Está en la página 1de 380
GRAPHS Exercise: bl I ' (0,0) (0,-2) (dy tem, 1 ax 2K when x~9 07, dd 54.0 ‘doe AY is not defined. Ge ¥ s+ Tangent at (0,0) is vertical. ®) Similarly, the tangent is vertical at (0,-2). 3Vxt when x99", dy —5 +20 Oe when, x05 a 3-0 be — DRG- Arnos J. dy is not de Finedat (0,0), aX » Tangent at(oe is vertical,alse The tangent at (0,2) is vertical, {0,20 is @ critical point} & Similarly, the tangent is vertical at (0,0)f critical point] when x — when x? -1* (he) is an angular point Levitical point J be S901 when x —70t doe Con) is om angular point 1. CONIS @ Lvitical point, where dy is not defined. Days ye 4 ( ded )x ie bee + tox BE FBR + 2% 2 OVE te 2x lm 1 AY 2 6NRE OK when x OF dx 1K + 44 is nor de Fined. dx J+ Tawgent at (0,0) is ver tical. dy 2 se + (- de xx SBE We. bio te ate ve JAY 2 6ve =e when xvot oe 2ve £. Tangent at (0,0) ix vertical. |The gradient at o- 49.2 when x7 oF ae 4Y. 0 when %2 07 ox #: (0,0) 13 an om (00,0) sa cer ar point, ‘cal point} (Hea) at) AY. 2H) when x9 07 dx BY. 2et when x9 0F ie 5. Tangent oF (00) iS vertical. oa) dx 33. Q when x» OF de The gradient at Oris 0. is -2, “3. ow a 2 » yrFcx) at > a ye $d yer ye foo x2 x— oF 3 At the points (0,-2) and (22), Eo is not defined since ot each of them He derivative tothe left isnot equal othe devivative te the right These 2 Crifcal points caw be alse called angular points. Exgrcise : 1-2 Co) ~ Exercise: 1:3 = Iw ixl 1 7 . oe | 2 ae psn be+ ee | 7 21 ye sive % = ae = a 1 ae { © i -t (a- Pee 2 I 5. "a ye sin 1H? ysl Fool I a 2.@) I ca] Go I 7 / \ - \ i \ysa-x! +: The graph is an even function as itis symmetrical about +he yraxis’ and Jor f(-x)= fcx) 6. 5. The graph is an even function as itis symmetrical about the y-aris aud for few fox) Tor” \ : Yes, itis an ode function because it is symmetrical about the origin, However, we can do it mathematteally : Fe) = oct sine m1 #Sin(-m) = fez) 4. Pox) is aw edd function. xe- Sinz, 6 AS foeye gle) + nex) \ fled = gf-2) + WE but gtr gle> Loieris an even function] hGd=h(2 Lhe is an even function) J. Fl2e) = ga + hex) i ftes # ex) !. fx) is an even function, Exerase: 14 Yes, itis an eddfuuction since itis Symmetrical about the origin (0,0). We can prove this also as Fede xsinx -F(-x) =-x sin(-x)= xsinx - $0-0= foo 2-f0c) is aw even function. 6. As FO= g(x) Wex) LAO gO he-x I bat 9 (-%)= gle) anol a a! WC = he) [since both 96x) and h(x) ave oud 2. £02900 hoy fted= £09, 1. ftx) isan even function I -1- VS Exercise: Yes, because it's symmetrical 1 about the y-axis. gids oh 1 gt-x2_t_ = (Ps) 3 aes ees = ++ ge) is an even function. | ! Dole! When taking reciprocal of a fumetion FOevery point of aaa wit z-axis becomes an asymptote for ye fs) Oe) = — + Fis ee Oe = © x 30 ‘+ glx) is an edd function and is symmetrical about origin as shown. note: When squaring a function F(x) every intey section with x-oxis be; wes minimum during ooint wy UeF PO nae Co-1) F mhen cabin: £00), every itty section wil Grane beeslrd fh Woceetaet webee ‘ inflexion for 4-Eheoy!, 2-@) x) (2,16) (216) ™ (64) yore)? cart) 3.@ us (sing) -4- Exerase: 1-7 % 7 -lo- i dy S ® I (a2) 8 24 \ @ CN \ ( 2% 0 an rx iw) “vt ad ; ce urs 4 sine ' (a When square rooting y=f00) every value of y less tran will nolon ger . exist ond every positive valueoP ywill be EXERCISE : 1-8 i Square rooted Fo qet u> JFea- eh j I ye Vase get N 1 i \f/N\ £ ! | of © 73m BN3 aaa \ I fa by uit cose Sy tN COR where yt 1 ye Veosx isthe part of the graph above Reavis and yIia eAN® FOr below satis 4 @ Ye 4 sing esi i 1\ A eln(sinx) t ye lu (sina) i -Ule Fare Ww (e*3) 1 Fe) = Iw E (-209% 3) = Im G3) 1 ft-x? = fed L. fc) is an even we can see that it ts symmetrical on the y-axis +0 further verify this property. meton. Hs0, ated fix) = 2 fe fege 2714 2"! Csince I-rel= I2el) J. fae £(-x) IF Ge) 18 an even function. Also, we can see that it is Symmetrical on the y-axis +e further verify this property. 5. f(x) = Of h(x) 2 fld= gf W(-n9) = 4 Lhe) (since h(x) is even 2 h(-x- h(x) Lf Feed LEG) is amen function. | Exe rose: 49 y My an. | x an sing ion iit Afferent dx hazytay | a 2 9*Q <0 1. dy. xt 2. The tangent at the critical point (on 18 horizontal. i The tongentat the critical point (40) is vertical, G72) xt yttxy=3 using implicit difPeventiation 2x +489 ey txdy co i (2ytx) dy 2 (2x4 y) 1 gs atu) e424 “The tangentat the critical points Cb2) amd Uy ove hor jrontal. “ThE tangent at the critical polwts (2 and (2,-1) ave vertical. ee N 4. con) xt thytey ° (2,0) (2:0) (0,-0 1. The versical tangents (-2,0) aud (t0) ave critical points. !. The vertical tangents (2,0) and (-2o) ave evitical points. R- Exercise: 1-10 = i * 2M b= kx The roots of this equation are the points of intersection of I yekx anal ya x'- Ue? afi From the graph we can see ff that theve ave 2 vea\ veots for the equation, one of them (0,0) I aud the other has co-ordinates x74, y70. 1 2 Gan! Oe (e474 ea 1 The voots of this eq uation ave the points of intersection of yek amd ye Cent xh From the graph we cam see thal thereave 2vtel veots for the equationlf when 0< 2 From +he gyaph the solution Shaded part ie x73 yee yet (by Ieosxl sg} Osx San cosx= 4} ov cosxe-t 3 for wens} , %. wsxe st vxe Dtzew ov xe- D420 for keo, x= 7 for Fe hee ST for cos + tonne cos 2 fe xe AE a 2am oy xe 28 4 eer for 22 for ket, ace $F : ution: ST 2 From +he graph the solution is | for sin2x=-3 , <. sin2xe sin(-Z) Te — = 36% . Zr2ew oy 2x +207 = tye -— 7 Ss s d ve 5 —— ea x Ss x S F — 7 3a) Txt + lyl=t ~1- ) -\ulel 6. MY ict tyler 1 Soe NoxY a =o] a - or ye Len) | 2) Ve. (adeby [/ 7 ) le. uh (eX te) o7 yz ble er) ~ The equation f(x)=K will have & voots when the horizontal line y>k crosses the cuvve & times. 7. -Lek) a= =¥)+i(9)~ ¥2)= (2-4) “(4 - 2) = 2-2 (©) 12 = (%x2— 02) 2 +a) = = (x22 yiy2)~ lary +21) = hh Omen Gitines Gente) (aee ahi +93 sty devi habeod xd+y} 1 x; j;—22 a a gta) stag 27% ‘aot x2 +2 *2tI2) + (zd) +i) +i) 4+ follows from (e) with z=1 and z) (1)_1 (@) Identity " 5 5 follows from (c) with zj=5 and z= Identity 52 5 Solution (a) Using the results in question 4 gives a(@) +ba+c=(aa")+ba+e=aa*+ba+c=0=0 (b) If @ is anon-real mamber, then Ima #0. Hence & # a, since Im(@) Thus if a is a non-real root of ax” +bx”-+c=0, where a, b, c are real, then @ is the other root of this quadratic equation (see (a)). 6 Solution (@) Im 2 =2 = c=x42i and 2? (7-4) +i(42), xeR real => 4x=0 = x=0, =2i. (b) Rez=2mz = z=2y+iy and z?-4i (4y?- »?)+i(4y?-4), yeR 2 -4i real => 4y?~ =0 = y=tl, ‘SD Solutions Series 7 Solution waaay Thus Z is imaginary = z is imaginary. 8 Solution (a) -25=25i, :. -25 has square roots Si and ~S5i. (b) Let (a+ ib) =-6i, a, BER. Then (a”—57) + i(2ab)=~6i. Equating real and imaginary parts, a*-b? = 0 and 2ab=-6. 2_9 @-+=0=>'-9=0 a (2-3)(a?+3)=0, a real => a=¥3, b= has square roots ¥3-iv3, —V3+iv3 © Let (a+ib) =i, a, BER. Then (a? -2")+i(20b) 3 or a=—V3, b= 43. Hence -61 j. Equating real and imaginary parts, a” ~b? =0 and 2ab=1. =O at 0 2 2 i ee aoa J (202-120? +1)=0, @ real => a=J5 b= 5 or an-ap, b 1 has square root Fe +1. ~ a (@) Let (a+ iby =—44+3%, a, beR. Then (a? }+i(2ab) = ~4+3i, Equating rea) and imaginary parts, a”-b? =—4 and 2ab=3. a2, = 49 4a" +160?-9=0 4a (2a?~1)(2a?+9)=0, a teal = a=—~, b= +e, b=, (242 =1)(24' a real =) a=Tp, b= of a=, J Hence ex taqeneros Joide, bid, (©) Let (a+b) =-5~12i, a, beR, Then (2? ~b")+i(2ab)=-5~12i, Equating real and imaginary parts, a? — 5? =—5 and 2ab=~-12. S= a‘ +5a”-36=0 (2?-a)(a? +9)=0, a real => a=2, b=-3 or a=-2, b= 3. Hence ~5~12i has square roots 2—3) 2-43: ‘78D Solutions Series 9 Solution - 3 (a) A=-3=37; 2 x= es af 4tiv8 44,1 = elt eG cs (©) Find A: 16(1+2i)° +16(3-4i)=0. Hence 4x? —4(1 + 2/)x -(3-4i)= 0 has two 1 equal solutions x= +i (@) Find A: 4(1+i)° - 40 = 32%. Find square roots of A: Let (a+ ib)’ =-32i, a, beR. Then (2-0) +i(2ab) = -321. Equating real and imaginary parts, a? -b? = 0 and 2_ 16 ab=-16. a ---=0 = a*-16=0 @ (2?-16)(4? +16)=0, a real => a= =-4 or a=-4, b=4, Hence A has square roots +(4—4i). Use the quadratic formula: ix? -2(j+1)x +10 201+) +4(1-i) 2 has solutions ~3i or x=3+i. 10 Solution (a) band c arereal, :. 3+2i is the other root of x? +bx-+c=0. Hence c= (3-2i)(3+2i) and -b = (3-2i)+(3+2i). Thus c=94+4=13 and b=-6. () Ima@=2 = a=x42i, xER. k real = %=x~2i is the other root of x? +6x+k=0, Hence k= (x-+2i)(x-2/) and -6=(x+2i)+(x-2i). | k=x?+4 and -6=2x. Thus x=-3 and k=13. Hence both roots of the equation are -3+2i. (©) Let z be the other root of x7- 3+ v) 2=(3+i)-(1-2i)=2+3i. Hence 1 2i)z = (1 2)(2 +31) = (24+ 6)+i(-4+3) = =8-i. ‘)x+k=0. Then 3+i=(1-2i)+z. ‘SD Solutions Series Exercise 2.2 1 Solution 3+ a+ 4 0 at at °. 6+ V(-4,-6) 2 Solution S(2,3) 243i ° 22=6-4i QG,2) 2ie=-4-6i . 2 3 6 x P(3,-2) ° T(6,-4) In each case P(a,b) represents the complex number z= a-+ib and @ is the principal argument of z { ‘SD Solutions Series @ (b) P(-1,v3) © @ P-L) z=2-i kl=v441 = V5 arg z= ~tan™'(1/2) TSD Solutions Series © wo @) (hy w 2 F P02) 342i 944 =V13 -— tari 1(2/3)=9 arg z= -tan '(2/3) ‘SD Solutions Series zai(itly=-iti el= Viet =42 st Ga n-$ arg c= 3 Solution Let 2=~2N3+2i P(-23.2) bl=viged =4 O=n—E = arg 2= 6 6 Sn Sa =4 cos + isin =4cis 6 € 7G) 0 4 Solution Let 5 =[2)|, 7% =|zo] and 6, =arg z,, 82 =arg z). Then 4 =n(c0s6,+isin®,), %=n(cos(-6,)+isin(-6,)), (-8,)+isin(-8,)}, 2 =7(c0s0, +isin®,), Z=r(cos| (@ a2 = H7(cos(@, + 2) + isin(0, + 8,)) => = 22 = H1(c0s(~(8, +82) + isin(-(0, + 8,)))} 2)) + isin((~6,)+(-2))} Therefore, 2,2 But 3-2 = n72(cos((-01)+(-8 (&) Let r={e| and argz =0. Then z= r(cos6+ isin), Z = r(cos(-0) + isin(—€)) (cos@+ isin@) and ). Thus ( z and + =1,(cos(-0)+ isin(~ z (coso + isin). Hence 5 Jee z ‘SD Solutions Series ©) AeA —62)+ isin(@, - 8 4a oof, 64) +180, ~03))>[2 ae n But @ (cos(—6, + @2)+isin(-, +02). Therefore, (2 5 Solution Define the statement $(n} {z"|=|z|" and arg(z")=nargz, n=1,2,K Clearly (1) is true. I $() is tue, then [2 4) v-fet"]=[eP* if 5(&) is true. and arg(z*)=kargz. Consider S(k+1). ke le{*-|z], if S(k) is true. arg(z**") = arg(z* -2) =arg(z*) + argz = kargz+argz. if S(k)is true. v-arg(z*") =(k-+I)argz, if (k) is true. Hence for all positive integers k, S(k) true implies S(k+1) true. But S(1) is true, therefore by induction, S(n) is true for all positive integers n. (2") =nargz for all positive integers n. 6 Solution ©@ |al=4>[]=4 = 64. aga = 4=>arg(z})=3-4=2. <2 has modulus 64 and principal argument 7. a2 Hat ® kale2ftfad. ‘TSD Solutions Series x 1 x angen = Gane eG has modulus + and principal argument —— \ zl ba 6g. =| 5 22 z2] are(z}) + al : a sa 2.4L has modulus 32 aid prineipal argument 2 7 Solution Let 2) =-V3+i and z)=44+4i.Then 2 feel © icin x ei)= = + isin =4V2, of 2 5H) i{ cook isin) | agen =7 @ (W3+ia+4r a fez]= 80/2 and +22. since sr, the principal argument of 7 (cos +isin7 =spcis 2%, aa 2 ae 2 ‘7SD Solutions Series u 8 Solution ange =. If is eal, then =a cos5 + isin =I 3 3. arg(2")= ker, k is integral. But arg(z")=nargz. Therefore n Gokn, k=0,41,42,K , on = 3k, k= 0142,K Hence the smallest positive integer n such that <” is real is 3. If z” is imaginary, then ag(2*) = oe kr, & is integral. But arg(z") =nargz. Therefore n 7 =e kan, k=08142.K, +3, k=08142.K Hence there is no integral value of m for which 2” is imaginary. 9 Solution are(i)=are(i)arge= 240. ‘SD Solutions Series 2 10 Solution B (a) Let 2=14V3i. Then 2=2 G3 11 Solution 3 Let z=V3+i. Then 2=2: (Bd) afconGian) = ¥3~i.Then z= and [y 2, angay =~argz=—%. Hence 210 = 1024, |a9| "| Sx arg(2'*)=l0arge =F = FR a-tn+E. Therefore 204710 = son oo) sisn(—8))s vor cox isin) =2 1024-cos =1024 3)) eB rin reish, B-i= aei(-2} (vB+i)° + (5-1)? = 1024 ‘SD Solutions Series 12 Solution i, 2y=3-4i,and 2=2—* then fa[=Va947 =512 and ai Let 4=7 argz, =-ar"(2), |ea|=V9416=5 and argz, =-un(f arg z = arg q — argz) = ao(4) tan (3). Use a well-known formula: a} =1, principal argument of 13 Solution nA +s) = a{cos%+ isin%), 31 . @=2V2B, where Breos E+ isin. zz can be expressed as 2 Bz—>2V2Bz.Let P, Q, Q represent z, Bz, 2V2Bz respectively. Then [be|=|0/-lel=[e)=> 00, = oP arg(Bz)= + argee ray OQ, makes the angle aa with ray OP . Hence B— Bz is a rotation anticlockwise about P through a and 2-9 Gz is the composition of this rotation followed by an enlargement about O by the factor 272 . ‘ISD Solutions Series u 2=3i, [ and argz= P(0,3) |Bo|=3 and arg(B:)=7= +5 |oco|=6V2 and arg(z)= © az=-6-61 14 Solution (a) Using the method of completing the square: 2 Since -2< p<2, x4 pxt1=0. (b) Using the quadratic formula: X+xtl=0= A=35 x5 P-Y5+1=05 Aa t= ‘SD Solutions Seri ; + 8 be 3 - &, are the solutions of the first equation, and ZCOA = arg x, ~arg.x; = ZACB = ZACO + ZBCO 1 But ZACO ~ ZAOC), since AO =0C and ZBCO= i (e- £800), since BO=OC=1. ‘Therefore ZACB =m — (£A0c+ £B0C)= x3 (0n- ZA0B) = AOB= 5. ‘SD Solutions Series 48 Solution (a) Using the quadratic formula: n24i2v2 2 aJ=Vie2 = V3 and argx, =2- tan V2, x242x43=0 A=-8>x= V2i. Let x, xy =-1~V2i. Then |x]= aess=-(e-t0"). * A(-1v2) Y B(-1,- V2) (b) Using the quadratic formula: —2pti2yg-p* OP op aslgaph ee xt42pxtq=0=A=4p?-4g>x 0, LHOK =F ang t= % when p<0 or argx3 = when p>0. In each case Lee must be equal to 1. Hence ZHOK is aright angle when q-2p* \P| : (ii) A,B, Hand K will be equidistant from O, if |x| =[x. |= V3 and |x3|=|z4|=/9. Hence q=3. ‘SD Solutions Series Exercise 2.3 1 Solution OA, OB represent z, 22. OACB isa parallelogram and ZOC represents z +22. Since |z;|=1 and |z9|=1, OA = OB. Hence QACB is athombus. Therefore ZCOB = + AOB. But ZAOB Thus %—arg zp and COB = arg(z, + z2)— arg za- (a) LetOA, OB represent z, z. Construct the parallelogram OACB. Then OC, BA represent z +22, 21-22 respectively. Since |z|=[zo], OA = OB. Hence OACB isa thombus. Therefore diagonals OC and AB of OACB meet at right angle. Thus BA is obtained from OC by a rotation anticlockwise (or clockwise) about O througt , followed by an enlargement in © by some factor k, then by a translation to its position and a diagonal. Hence 4-2 =hi(z, +22) (Or 2-22 =—ki(z + 2). In either case, the number At? is ane imaginary. ‘SD Solutions Seri ‘SD Solutions Series 9 (b) LetOA, OB represent z,, zp- Construct the parallelogram OACB. Then OC, BA represent 2, +22, 21~ 2 respectively. Since arg(z, ~ 22) =arg(z; +29)+, BA is obtained from OC by a rotation anticlockwise about O through &, followed by an enlargement in O . Therefore diagonals OC and AB of the parallelogram OACB meet at right angle and OACB is a thombus. Hence OA = OB and |z1|=|z2]- Let OA, OB represent 21, 22. Construct the parallelogram OACB.ThenOC, BA represent 2, +22, Z1~Z2 respectively. Since |z, + zq|=|z, - za], OC= AB. Hence OACB is a rectangular. Therefore ZAOB =, But ZAOB = argz,—argz (or ZAOB = arg z, ~ arg z,). Thus arg t) 2:2. wa) 2 4 Solution Let R represent iz. We know that the transformation z > iz corresponds to a rotation anticlockwise about O through the angle % in the Argand diagram. Therefore OPOP is a square. Hence OPQ is a right- angled triangle. 5 Solution OP, OQ represent z,, 2). Since OPQ is an equilateral triangle, OP = OQ and - Hence OQ is obtained from OP by a rotation anticlockwise (or ZPOQ 3 clockwise) about O through %. Therefore z) = cz, with a= cos%+isin® (or acof-f)=in(-§), P+ oe-(1402). But t+0? =a. Hence 2f +2 =a2? = 2 (0a) = 22. 2 Breda. 6 Solution al =|e1 + Z2|- Let now z #0 and z) #0. Then fal If 4 =0 or = \zo|= |e, +29] with equality if and only ler + 22 — 2a|—feo] S fey + 22] if atm=k-(-z), k>0. 1+ Zp| with equality if and only if z = {za + z}| with equality if and only I+k)zp, k>0. len +21 -2il—[ey lel- if ata =k-(-y), &>0. Tape k>o. feal=leilslei +29] with equality if and only if 2 = ~kzq, k>0, oF 4 =0, or lz) + z9| with equality if and only if Hee Jala 2=0. ‘TSD Solutions Serie 2 7 Solution latals| \zo|=25-+6=31 and this greatest value of 31 is attained when z2 = ky for some positive real k . But |zq|=6 and z2 = ke => 6= 25k. «-|z, +29] attained the greatest value of 31 when 2) = (24+ 7i)= 188 +23. ker +z2|2Ja+lzaf= 25-6 =19 and this least value of 19 is attained when zp = —Kz; for some positive real k. But |z9|=6 and z2 =—kz, => 6 = 25k. + Z2| attained the least value of 19 when z, =~ (24+7i) Mt 4 2 25 35! 8 Sotution ‘We shall use the method of mathematical induction to prove this inequality. Define the statement $(n}|z, + 22+L +2,|$|zi|+[z2|#L +eq) 2 = 2,3,K, Consider S(2) [zy + za|<|z 2| => S(2) is true. Let k be a positive integer, k> 2. If $(&) is true, then ler zt +z4| Sey] +feofHl +]z4]- Consider S(k +1). fey + zal zy + eesil S[(zy + Z24L +24) + Zeal Ser + Ze 4L +241 + [Ze (triangle inequality $(2)) |zy|+|zo}+L +{zi|+|zeail. if S(k) is true. Hence for all positive integers k (k 22), S(k) true implies $(k+1) true. But S(2) is true, therefore by induction, S(n) is true for all positive integers n 22. & fey + 22th +24] Sli] +[zz}+L Heql, for all positive integers n 22. ‘ISD Solutions Series 2 9 Solution AQID = AOBA, 2D _ 01 | OA OB (al |e] ZDOI = ZAOB = arg z5 = arg 2, — arg z2. Hence mc Fel foal EAL and args = arg ~arezy (1,0) a UR z3=41 and OD represents the quotient of z, ae and zp. SD Solutions Series 2B Exercise 2.4 1 Solution Using De Moivre's theorem 2”° =2!° cis(5n), (2) = 2! cis(-Sn). Now 2Re{z™)= 2!" cos(Sn)=—2048. Hence 204 (z= 2 +(2) (144 +(0-1" 2 Solution Let =-14 Vi. Then z=o{-448i -1-3i 2" =2"(cos 24% + isin 2a"), Now 2° +(z)" = 2" +(2 iY" 22" coef 20% Val =2*coo 228), \" \" 6mn If n=3m, (- (1 -. 22" cos OT) _ amet on (ctf +(-1-Vif = 2" cof SU) tem= Smt, (146i) + (1 Vi) =2™ co an (cos 2x + isin 28) and 2(cos28-- isin 28). Using De Moivre's theorem 2Re(2")= 2" cof 228), Thus (~14-3i)" + ‘SD Solutions Series 24 3 Solution 25 =1=9|2|=1. Hence Sth roots of unity have modatus 1 and their representations P, (k=9,1,2,3,4) lie on the unit circle with the centre in the origin, By De Moivre’s theorem one root (Zo) has argument zero, the others being equally spaced around the unit circle in the Argand diagram by an angle 2%. Hence the complex Sth soots of unity are 1, cis(28), cs( 42). Since 2P,0P, 4, = 28 and OP, = OF 41 P,Prat = 2sin¥ for any k =0,1,2,3,4(P:= Ry). Therefore the points % P, (k= 0,1,2,3,4) form the vertices of a regular pentagon of area $sin2E (=5-(area of AR,OR)) and perimeter 10sin® (=5- 7A). 4 Solution }i]=1 and arg(~1) =. Hence the complex Sth roots of -1 all have modulus 1 and by De Moivre’s theorem one complex Sth root of -I has argument £, the others being equally spaced around the unit circle in the Argand diagram by an angle 2%, ‘Therefore the complex Sth roots of -I are cos isin, cos3&+isin3®, and -1. Then 25 +1=(¢+I)(z~cis$)(z~ cis(-$))(z—cis3#)(z—cis(-28)). But (z~cis)(2—cis(-8)) = ((z-cos4)- isin) (z~cos)+ isin) =(z-cos3) +(sing —2zcos%F +1. 2 x —cis3®\z—-cis{-32) ~2eeos$e1 and (z—cis3#)(2—cis(-28 2 B+1=(z+I(z?-2ze08$+iz? - 2200548 +1). ‘SD Solutions Series 2s 5 Solution By De Moivre’s theorem and 2” =cosn0+ isinn® and 27" =cos(—n®)+ isin(-n®)=cosn® —isinn®. Then 2”+2-" =2cosn@ and w= =2isinnd. a 4, -1\* (a) 2cos0=2+2"!, Then 16c0s*6=(z+z"') . But (ere"acteaeoeartert a(e+2 }+4(z? +2") +6. Hence 16cos* @ = 2cos40 + 4c0s26+6 and cos*@=1(cos49+2cos26+3). (&) 2isind= 2-2". Then 3245 sin @ =(z- x) -But ~523+10z-102" = 25)-2-29)s10fet-2" Hence sin°@ = (sin 50—Ssin36+10sin@). isin 58 — 10isin 30+ 20isin® . 6 Solution The cube roots of unity satisfy 2-1=0. But z?-1=(2—}(z? ++). Hence @ z=lz?424153 (b) z#l=e 2? +241=0. 7 Solution @ =1. Since @ is a non-real root of unity, @” + @+1=0 (it follows from the factorization «* -1=(0-1)(u? +0+1)), Let =(1+30+0)) and n=(I+0+30") Then 41=(1+0+0?+20) = (of (since 1+0+07=0) =40? and n=(I+0+0?+20")' = (20%)? (since 1+0+07 = =404=40 (since @? =1) Hence 21+ 22 =4a? +40 =4(0?+0+1)-4=-4 (since o?+@+1=0) and 44:2, =44-4o=160 =16 (since @? =1) ‘SD Solutions Series 26 8 Solution (@ 2=v3+ s+ 2 {2 x 2 cos=+ isin (3 s.[el=2 and argz=%. By De Moivre’s theorem one square root of z has modulus V2 and argument Hence the two square roots of z are +V2cis. 2-2 (4,-1.3) = 8{cos(— in(—38) (b) ¢=-2-2i= 23(-4,~ 451) =8(cos(—2) + isin(29)). o-|]=VB8 and argz=-38. By De Moivre’s theorem cube roots of z have modulus ¥2 and arguments —$+ 48, k=—1,0,1, Hence the three roots of z are Picis-§), VBcis(—U8), V2 cis 2) 7SD Solutions Series 1 Exercise 2.5 1 Solution 2 Solution ‘TSD Solutions Series 27 Let z=x+iy. Then Z tty’, zti+le x+yts2xtles (x-1)'+y?=2. Hence P lies on the circle with centre (1,0) and radius V2 Let A represent -i and B represent ~i. Then, if P represents z, AP represents z+i and BP represents z+1.Hence AP<2 and BP makes an angle between O and 4 with the positive x- axis. 3 Solution Let A,. B and Q represent 1, i, z respectively. If _ le-ils[e-i,then AQ = BO / “ and the locus of @ is the 'p(3,2) perpendicular bisector of AB. x + Since AB has midpoint (1,4) “ 77 PON and gradient —1, the locus of © passes through (4,4) with gradient 1 and has Cartesian X equation y=x Let C represent 2+2/. If |z-2-2i$1, then CQ=1 and @ ties on or inside the circle with centre (2,2) and radius 1 Let now |z~1}<[z—a and [e-2-2<1. Then AQS BQ and CQS1. Hence @ lies on the right-hand side of the perpendicular bisector of AB inside the circle centre C and radius 1, or Q lies on the boundary of this region. If P describes the boundary of this region and arg(z—1)=, then CP=1 and AP makes the angle = with the positive x-axis. Thus we must solve simultaneously two Cartesian equations (x-2)'+(y-2)? =1 and y= x—1. Substituting the second equation into the first gives (x-2)° + (x~3)' <1 2x?-10r+12=0— =23=9 y=l (when x=2), (when x=3). Therefore such P represents z=2+i and z=3+2/ | ‘SD Solutions Series 29 4 Solution Let A represent -3. Then AP represents z+3. AP has gradient tan()= V3. Hence the locus of P has Cartesian equation y=V3x+3¥3,x>-3.Now OP =[z|. Hence the minimum value of |2| is the perpendicular distance from (0,0) to the locus of P. Therefore the minimum value of |z| is doing 28, Since AP has gradient tan = V3, OP has gradient 1 51 a 3 wan(2) when |z| takes its least value. Hence modulus of z is Es and the argument of z is = when «= Ma) 33, TSD Solutions Series 5 Solution Let A represent 1. Then AP represents z—1 and AP=1. Hence P lies on the circle centre A(1,0) and radius 1. Let @=argz and B represent 2. Then ZPOB=6 and 2PAB = arg(z—1). But ZPAB =2ZPOB and arg(z”) = 2arg z. Therefore arg(z—1)=20 = 2argz= (2?) 6 Solution Let P(x,y) represent z= x-+iy. Then zci_ xti(y-T _(+O-)(e+))-9) z+l (x4l)+iy ~. if 2—+ is purely imaginary, then +1 x(x+1)+ y(y—1)=0. This is the equation of the circle with centre (. radius Jp. and 7 Solution ‘7SD Solutions Series 31 Let P(x, y) represent z=x+/y. Then aD i aa xtiy— z xtiy x+y’ ‘Therefore the locus of the point P has Cartesian equation x=0(y #0) or x+y? 8 Solution Let A(2,0), B(-2,0) and P represent 2, ~2, and z respectively. Then AP and BP represent z~2 and z+2 respectively, andarg(z~2)=arg(z+2)+4 requires AP to be parallel to the vector obtained by rotation of BP anticlockwise through the angle of & . If P lies befow the x-axis, AP must be parallel to a clockwise rotation of BP. This diagram shows arg(z-2)= arg(z+2)-3. Hence P must lie above the x-axis. ‘SD Solutions Series 32 Since alternate angles between parallel lines are equal, ZBPA=% as P traces its locus, Hence P lies on the major are AB of a circle through A and B. + The centre C of this circle lies on the perpendicular bisector of AB, and the chord AB subtends an angle 2-2=28 at C. a Therefore OC = and AC =f. Thus the centre of this circle is C}0,-4| and the is radius M3) ie % 9 Solution Let P(x, y) represent z=x+iy. Then ‘TSD Solutions Series 33 10 Solution Let P(x,y) represent z= x+iy. Then x-+iy =r(cos® + isin@)+1(cos®—isin®)= (r+4)coso+i(r—4)sino, =(r+4)cos@ and y —4)sino. 4x? ay? Hence —+=2-=1 sind ee Dt (a) x=4c0s0 and y= 1) 45. Hence x?- ) xa(r+4e and y + cy PY 4x? 4y? 2 (b) (a) whe * So ‘SD Solutions Series Diagnostic Test 2 4 Solution (a) (i) y+z.=(2+i)+i=2+2i Gi) +22 =(44i)+(2+3/) = 644i () @ 4-H=(2+i)-i=2 (i) q-z=(4+i)-(2+3i)=2-28 (©) @ yz_= (24s 2+? =-1427 (ii) 2) = (44+1)-(2+31) = 843? +1214 215414 oor. +i (4+if (8+3)+(2-12) 243)” (2312-3) aoe 2 Solution (a) (i) Re(3)=3 (ii) Re(4i)=0 Gil) Re(B+4i)=3 (b) (i) Im(3)=0 (ii) Im(4i)=4 Gii) ImQ+4i)=4 © @ @)=3 Gi) (4i)=-4i Gi) B+47 3 Solution (x+iyf =3441>(x7-y*)+Qm)i=344i Equating real and imaginary parts: x” ~ y? =3 and 2xy =4 xt—x?y? =3x? and x?y? =4 Then x*= 327-4 = 09 (x?-4)(x? +1) =0, x real, . x52, ySlorx=-2, y= ‘SD Solutions Series 35 4 Solution @ As4=4? =x (e) Find A: A=(2-i) +8i=3+4i. Find square roots of A: Let (a+ib)' =3+4i, a,beR. Then a?—b? =3 and 2ab=4. 2. at — a? =3a" and a*b? Thus at ~3a?—4=0=9(a?—4)(2?+1)=0, a real, . @=2,b=1 or a=-2,b=-1. Hence A has the square roots 2+i, -2~i. Use the quadratic formula: x?+(2~i)x-2i=0 ns soins x= 22-4) siorx 5 Solution (a) z=2-(cos0+isin0) = |z|=2, argz=0 (b) 2=21=2(cos$+isin§) © zat4Vii=2b+i8)= @ c=--i=a-4-4)) 6 Solution (@ ea-asi=Ji-+ fi} = Vis ) z=1-t=J3(f~-fi)= V2eis(-4). ‘SD Solutions Series 36 7 Solution @z = 4cis(28) = a{— (b) z= 2cis 8 Solution fu |=2 and arg z (@) |zz0l=[eil-lza]=2V2 and arg(z,z9)= arg 21 + arg z2 {21 22] lal _ 2 ae (b) a lal 4 anf tree agen cc} 9 Solution zal+inv2-(+ik)=Vieisg, V2 and argz = ¥. Then |2""|=||° =(v2)° =32, kd arg(z'°) =10argz=10 =58. But 58>. The principal argument of z'° is 3E-2n=8 10 Solution (@) z=lti ©) © ait? =-14i (@) z+1=2+4i © ‘SD Solutions Series 37 11 Solution (i) Let OA, OB represent 2), 22. Then (a) OC represents 2, +z, (b) BA represents 2,2 (ii) (c) AB represents 22 —z. 12 Solution By De Moivre’s theorem: (cos@ + isin@)* =cos(40) + isin(48) = cis(48) 13 Solution By De Moivre’s theorem: cos(20)—isin(20) = (cos0 + isin®) cis)”. 14 Solution By De Moivre’s theorem: (cos + isin®)” = cos(28) + isin(26). But (c0s0 + isin)’ = cos? 6 +2icos@sin8+i?sin?@ = (cos? @—sin?6)-+i2sin@.cos0. Equating real and imaginary parts we obtain cos(28) = cos?@—sin?@ and ‘ISD Solutions Series 38 sin(20) = 2sin ®cos®. Hence sin(20) __2sinOcos@ _ tan(20)= <5 5(28) ~ cos?@—sin?6 45 Solution i 2y2eis(§) 8 cs. [e|=8 and arg z= 4. By De Moivre's theorem, one square root of z has modulus 22 and argument £. Hence the two square roots of z are £2V21 16 Solution YA Let g=x+iy. Then 2ax= i (a) Rez (b) Imz=1= y=1 2 > yA oO 4 ‘Let P represent z. Then (x-2" Hy =4 © oP |= 29 P lies on the circle, center (0,0) and radius 2. (d) Let A represent 2+i. Then AP NY X represents 2~(2+i) and |e-2-d=2= AP=2, P lies on the circle with the centre SD Solutions Series 39 (©) The gradient of OP is tan(-4)=—V3.. The locus P is the ray Vz, x>0. () Let 4 represent ~i. Then AP represents z+i. AP has gradient tan(2#)=—1. Hence the locus of P is the ray y=-x-1, x<0. 17 Solution Y + yer @| 2) is the perpendicular bisector of OA. argz = isthe ray y=x, x>0. argz=— isthe ray y=-x, x>0. ‘TSD Solutions Series (b) |e|=1 is the circle, centre (0,0) and radius 1. arg z =0 isthe positive x-axis. arg z= is the 2 positive y-axis. ‘SD Solutions Series 4 Further Questions 2 1 Solution Let 2 =3+2i and z)=5+4i. Then 2p = (3+ 215+ 41) = (15-8) + (12+ 10)=7 +22, A = (3- 215-41) = (15-8)- (12 +10) = 7-221. Hence |zjzl?'= 7? +227. But [zzal” =|ei)*-[zal” = (3° +2")(5? +42). Therefore P+ 22? = (3?-+2*)(5? +47) 2 Solution Hence atya(S+2)-(2+2). pu +2)=1 and a, b are real. Equating real and imaginary parts: 3 Solution Substituting x=1+i, (I+) +(a+2i)(1+i)+(5+i8)=0, (1-1)+2i+(a-2)+ i(a+2)+5+ib=0, . (a+3)+i(a+b+4)=0, a,beR. Equating real and imaginary parts: a+3=0 and a+b+4=0. Therefore a=-3, b=-1. ‘7SD Solutions Series 4a 4 Solution Let z be the other root of the equation x?4(I+i)r+k=0. Then z+(1-2i)=~(I+i) and z-(1-2/)=k. Therefore za-(i+i)-(l-2 (-2 + i)(1-2i) = (-2+2)+i(4+1) = 5i. Hence k=5i and equation x? +(1+i)r+k=0 has roots x=1-2i and x=-2+i -2+i and k= 5 Solution Let %, Zp are the roots of the equation x? +(a+ib)x+3i=0. Then 2} +(a+ib)z, +31=0 and 23 +(a+ ib)z, +3i=0. But zf +23 =8. Hence 8+(a+ib)(z, +z))+6i=0. But 3 +2) = (arity =8+6i, abeR (a+ ib). Therefore 8~(a+ib) +6i=0, Thus (a?— 4") +(2ab)i=8 + 61. Equating real and imaginary parts, a*—6*=8 and =8=9a*~8a?-9=0. (a?-9)(a?+1)=0, a real. b=1 or a=-3, b= 6 Solution Find A: A=4?—4(1~4i)=12+16i. Find square roots of A : Let (a+ib) =12+16i, a,b€R. Then (a?—2?)+ (2ab)i=12-+161 ‘Equating real and imaginary parts, a?-b? =12 and dab = 16. a? - 4 =12=9 a 1207-64 =0, (22-16? +4)=0, a real. a . a= |, b=2ora=-4, b=-2.Hence A has square roots 4+2i, -4—2i. Use the quadeatic formula: x*—4+(1—4i}=0 has the solutions x = 4#(4+2!) x i orx=44i ‘ASD Solutions Series 4B 7 Solution 2 =2i=2AcosE + isin), +i) = 2fcos$-+ising), and argz, =. OP=0Q and OPRQ isa rhombus. Therefore ZPOR= ZQOR. Thus arg(z; + 22)=4(argay + argzq)= #8. Since diagonals OR and QP of the rhombus OPRQ meet at right angle, arg(z ~ 22) = arg(zi + z2)+$= YE. anole. + 22)= $8. ages —22) =HE. 8 Solution If 4-2) +23 —z4=0, then 2-2 = %4~ zy. But BA represents 47%, CD represents 24-23. Therefore BA and CD are parallel. On the other hand, z,~ 24 = 22-23. But DA represents z,-z4, CB represents z,~z3.Hence DA and CB are parallel. So we proved that ABCD isa parallelogram. ‘SD Solutions Series If 4 +izg ~ 23 ~izg =0, then i(24— %_)- Hence the 4% diagonals CA and BD of the parallelogram ABCD meet at right angle and CA= BD C ‘Therefore ABCD isa square. 9 Solution 1 1 a Hence + is real. Since ver Peg zw+z 2Rez er i 1 Rez=rc0s8, ==> : +r? 2rcos® Noting r? = 2Z 10 Solution - But x° The cube roots of unity satisfy x° — =(x-1)(x?+x4+1). Hence o¢ 1-90" 4a41=0, Cleary, @? =1. Therefore (140) = (0)? = (0°)! =1 ‘Then Hence (1a) i 9 (oe off -o*)(I- “"y -o')= 1-w-a? +0°) =(2-@-07)) ‘SD Solutions Series 45 11 Solution The cube roots of unity satisfy z equations z7-1=0 and pz>+qz+ if z=1, then p+g+z=0; . Therefore, if z is a common root of the then z is one of the cube roots. Thus if z=0, then pa’ +qo+r=0; if z=07, then pol? +qo?+r=0. Hence (p+q+r)(pa° +qa+r\ pa! + qu? +r)=0. 12 Solution (2-128 +23 +1). Therefore, if 26+ 23+1=0, then 2? 0. Hence the roots of 2° +z?+1=0 are among the roots of z?-1=0. Let z=cos@+isin@ satisfy z° =1. Using De Moivre’s theorem, cos(98)+ isin(98) =1+0i + c0s(98)=1 and sin(98)=0 2. 90=2nk, k integral. 25k, k integral. =2Ek, k=0,1,K,8 gives 9 distinct numbers z with argument 22k. ‘Taking If 2 +z3+1=0, then 2? =1 but 2? #1. Hence the roots of 2°+2? +1=0 are cos(28k) + isin(2Fk), k=1,2,4,5,7,8. < 26+23+1=0 has the roots cis(+28) cis 43 cis(t 4). ‘SD Solutions Series is the circle, l= center (0,0) and radius 6. |z~5=5 is the circle, center (5,0) and the radius 5. Since y- axis is a tangent line to the circle |¢-]=5 at point (0,0), if {¢|<6 and |z-3}<5, then ~S27 +y? =36, and \e-=5=9(x-5)P +9’ and {z~5]=5, then both x? + y? =36 and 6. - Hence, if z such that both {e x? + y? 10x +25=25. Therefore 10x 18 = 2 y=hy36-(I8 Hence the values of z for which both |z|= 6 and |z—5|=5 are 18 j2¢ 14 Solution (2) Let A represent 2. Then AP represents z-2, and \el=|z-4=> OP = AP. The locus of P is the perpendicular bisector of OA. Therefore the locus of P has Cartesian equation x =1 JSD Sotutions Series a7 (b) Let A(2,0), B(-2,0) represent 2, -2 respectively. Then AP and BP represent 2-2 and z+2 respectively. arg(z -2)=arg(z+2)+4 requires AP to be parallel to the vector obtained by rotation of BP anticlockwise through an angle of . yA If P lies below the x-axis, AP must be parallel to a clockwise B(-2,0) ° 4(2,0) X ‘tation of BP. This diagram age 2) ~ shows z x Das arg(z—2) e(z-2)=are(z+2)-F. o-2 Hence P must lie above the x- 242 axis. N vob yA Since alternate angles between parallel lines are equal, ZBPA=# as P traces its locus. oa : Hence P lies on the upper are 2 \e-2 AB of acircle through A and yA ‘The centre of this circle is the centre of diameter AB . Hence the locus of P has equation Let z=x+iy satisfies both |z|=|z~2| and arg(z~2)=arg(z+2)+. Then x and y= V4—1=3.Hence z=1+iv3. ‘TSD Solutions Series 7SD Solutions Series Worked Solutions to Popular Mathematics Texts Suggested Worked Solutions to “4 Unit Mathematics” ( Text book for the NSW HSC by D. Arnold and G. Arnold ) Chapter 3 Conics 47104 Bog Xo, (COFFS HARBOUR SENIOR COLLEGE AE Solutions prepared by: Michael M. Yastreboff and Dr Victor V. Zalipaev 7SD Solutions Series Worked Solutions to Popular Mathematics Texts OX page Exercise 3.1 1 Exercise 3.2 10 Exercise 3.3 16 Exercise 3.4 24 Diagnostic test 3 30 Further Questions 3 37 Solutions are to “4 Unit Mathematics” (by D. Amold and G. Amold (1993), ISBN 0 340 54335 3 } Created and Distributed by: ‘SD (Information Services) ABN: 13009821 © 780 1987 ‘The 7SD team welcomes all feedback. Correspondence should be addressed to: ‘7SD atin: Michael Yastreboff PO Box 123 Kensington NSW 2033 Exercise 3.1 1 Solution (a) The locus of a variable point P(x,y) is the ellipse with focus at S(10), directrix. m:x=9 and F 1 eccentricity e= 5. Let M be the foot of the perpendicular from P to m. Then M bas coordinates (9,y). 1 3 x(i-$}« 7 9-1. 9)” . ‘Therefore the Cartesian equation of the ellipse is a PS=e Pu e-oy=[ ) (x-9) (b) The locus of a variable point P(x,y) is the yh ellipse with focus at (0,1), directrix m: y=4 and 4 eccentricity 3 Let M be the foot of the 2 perpendicular from P to m. Then M_ has coordinates (x4). 5(0.1) PS m0: PM = x8 +(y~0? =(2} o-4) a] neil x+y fi--|=4-1, r( i) ‘Therefore the Cartesian equation of the ellipse is 7SD Solutions Series P(x y) (c) The locus of a variable point P(x,y) is the hyperbola with focus at S(9,0), directrix m:x=1 and eccentricity ¢=3. Let M_ be the foot of the perpendicular from P to m Then M has coordinates (1,y) PS =e: PM => (x-9) +y? =F (x-1)? (1-9) +y? = 9-81 ‘Therefore the Cartesian equation of, the hyperbola. is (A) The locus of a variable point P(x,y) is the hyperbola with 5(0A), focus at directrix and eccentricity e=2 Let M be the foot of the perpendicular from P to m. Then M has coordinates (x.1). PS =e: PM => x +(y-4) o-) x’ +y?(1-4) =4-16 Therefore the Cartesian equation of the hyperbola is 7SD Solutions Series 2 Solution a=5,b=4b (43,0) directrices: x (b) eccentricity: foci: directrices: ‘TSD Solutions Series (@) x42y? <4 2 FY oy ane a=2,b=W2=>b (42,0) directrices: = x=i2V2 ‘SD Solutions Series 3 Solution eccentricity: foci: directrices: asymptotes: (b) eccentricity: foci: directrices: y= asymptotes: ‘SD Solutions Series foci: (4ae,0) = (46,0) directrices: a xettopxst asymptotes: yetbewy a (a) eccentricity: $(2V2,0) 1+ foci: (20,0) = (#2V2,0) directrices: =exedv2 Pee asymptotes: y= ‘SD Solutions Series 4 Solution (a) We have the eccentricity end and the foci (44,0) of the ellipse . But the coordinates of the foci are (4ae,0). Therefore a=4 3 Then b? =a?(1-e*) =25: (1-)=9. Hence the Cartesian equation of the ellipse and the directrices x=29 of the ellipse 3 =1. But the directrices have equations x= +". Therefore a= @ 3 ‘Then 6 =a?(I-e?) = 36. (-4) =20. Hence the Cartesian equation of the ellipse ay s+ 36" 20 § Solution (a) We have the eccentricity e-3 and the foci (45,0) of the hyperbola 4 29 a bi But the coordinates of the foci are (ae,0). Therefore a =5: Then Beate —yai6(B-1}q9. Hence the Castesian equation of the ey? hyperbola is = yperbola is >= 2 (b) We have the eccentricity =3 and the directrices x=24 of the hyperbola 1, But the directrices have equations x=+". Therefore a=4 3 e aay Then b? =a*(e* -1)=36. (@ }=4s. Hence the Cartesian equation of the SD Solutions Series xy hyperbola is —--2— yyperbola is S2— 6 Solution ‘The equation of the ellipse is ~+2>=1. Thus we need to find the parameters and 6. Since the foci are on the x-axes, their coordinates are (+ae,0). Therefore the distance between the foci is 2ae=4. The equations of the directrices are a e a Hence the distance between the directrices is 2-— e 16. Thus we have two equations a ae =2 and’S = 8. From the first equation we get e Substituting the expression 16. Therefore a=4 and e for the ¢ 10 the second equation we obtain ; 1 Then 6? =a?(1-e?) =16 (-4} 12. Hence the Cartesian equation of the ellipse 7 Solution Let m and m’ be the directrices 4 m Im of the hyperbola Then for P on the curve, both Mh /p PS =e-PM and PS’=e-PM’, P where M and M’ are the feet of the perpendiculars from P tom S“(—ae,0) S(ae,0) *Y and m’ respectively. Therefore _ 0| 2 |PS-PS'=e|PM - PM'|=eMM’ ‘Thus |PS— PS" For the hyperbola als ‘SD Solutions Series a=3. Hence |PS- PS'|=6. Since b? =72, e= 3. Therefore the coordinates of the foci are (+9,0) (a) If PS =2, then |PS’-2|= ~ We see that PS + PS’=10. But MM’ = 18. Hence there is no such point P on the hyperbola, (b) If PS =8, then |PS’~8|=6. Thus PS’ 4. SD Solutions Series Exercise 3.2 1 Solution x (a) Cartesian equation of the ellipse is — Hence a=4 and b=3 16 9 Therefore the ellipse has parametric equations x=4cos@ and y=3sind, -n1, PS=a(esec@-1) and PS’ =a(esecO+1). Therefore PS— PS’=-2a. If P lies on the left-hand branch of the hyperbola, then -<0<-F or F<0sn Since for hyperbola e>1, PS=~a(esec@—1) and 75D Solutions Series B PS’ = —a(esec@ +1). Therefore PS — PS’ =+2a. Hence |PS~ PS'|=2a, 7 Solution (a) POQ is a right-angled triangle. Therefore OP? +0Q" = PQ’. a? cos 0+5 sin? 0+ a? cos’ +b? sin? ¢ = =al a? (cos8 ~ cos$)* +57 (sin@— sing)? Then 0 =~2a? cosOcos — 2b? sin@sing 2 o.tan@ tan = (b) PAQ is a right-angled triangle, ‘Therefore AP? + AQ? = PQ?. a” (cos@— 1)? +b? sin? @+ a? (cosp—1)? +b? sin? 6 = al a” (cos® cos)? +47 (sin®— sing)” Then -2a* cos0 + a? - 2a” cos +a? 2a? cos®.cos}— 2b” sinOsino P(acos®,bsind) ‘Qacoso,bsing) P(acos®,bsin®) A(a.0) Q(acos,bsing) ‘TSD Solutions Series > 8 Hence tan=tan2 = 8 Solution (a) POQ is a right-angled \ (o(asec ,btan 6) triangle. Therefore OP? +09? = PQ* a” sec? +b? tan? 0+ a ec? +b? tan? a’ (sec 8 - sec)? +5°(tan 0 — tan)? Then = —2a? sec Osec — 2b” tan O tang e.sinOsing = PAI ht-angled () FAQ ‘is a rightangled a sec8,btan6) triangle. Therefore AP? + AQ? = PQ? a(secO- 1)? +b? tan? 6+ a? (seco 1)? +5? tan? = a? (sec — sec)? +b (tan — tan)? Then QAasecd,btand) ~2a? secO+a*~2a’ seco+a? = ‘ 2a” sec sec —2b? tan Btand 4 cos@ +05 ~1—cosBcos = —;sindsind, ‘7SD Solutions Series 1s 9 Solution (a) If PQ is a focal chord through S(ae,0), then 101252) = onf 222), . 8 8 Expanding both cosines gives (e= Neos cos “er Dsing sing. Hence tanS tan = 122, Similarly, if PQ is a focal chord through S’(-ae,0), Then e g_lte replacing e by -e, tan tan ws (2.nP)= o{ Sasce$ stn). Also b? = a?(e? —1) 2.P has parameter Let Q has parameter 9. Hence tanZ tan’ = or tnZ tan? 142, 6 2 6 2 1-2 el 9 a3. stant =, tan = -3y3, _ 2 and tang = O has coordinates (/3see9,3tan@) = O(2V3,-3V3) or f-48 28), SD Solutions Series 16 Exercise 3.3 7 Solution (a) The tangent to the ellipse — 1 at the point (3,2) has equation 15" 10 2y 5. The eG 2y = al a+ 7g == x+y=5. The normal to the ellipse 7=++5 has equation = 10y 15-109 x-y=1 3x 15 at the point (3,2) 2 48 => = ‘The tangent to the ellipse = at the (b) x? + 4y 16 12 16 . 2. -3; point (2-3) has equation i _ 1=3x-2y=8. The normal to the ellipse 16x 12y 1 at the point (2,~3) has equation — 16-123 2x+y=1 (c) The tangent to the hyperbola ~--2-=1 at the point (3,2) has equation ‘YPC 8 q 2 32 _2Y 152x-y=4. The normal to the hyperbola = AZ =I at the point 3.2) has equation & +8 x¢2y= 2_ ay? x ane, (d) 9x*-2y? = 18 => =--* = 1. The tangent to the hyperbola =~ 2— (d) 9x? -2y oeeao le tang hyperbola 25 at the point (2,-3) has equation & = => 3x-+ y=3. The normal to the hyperbola z 1 at the point (2,-3) has equation 2422 =249=5 x—3y=11 29 : ees le 2 Solution (a) The tangent to the ellipse x = 6cos@, y= 2sin@ at the point where o=% has cos= ysin= xeos= ysin= 6 6 equation = V3x+3y=12. The normal to the ellipse 2 ‘78D Solutions Series ” x=6c0s0,y=2sin@ atthe point — where o=F has equation St _ 29 236-4=93x-V3y=843. cos= sin = 6 "6 (b) The tangent to the ellipse x= 4cos®, y=2sin® at the point where @=—~ has 4 eof 7) yan) ‘equation pp tye i The normal to the ellipse m 4088, y=2sin@ atthe point where has equation 4x 2Y 6 4 2xey=W. rk xsec= yians a 1=>3x- 3y=3. The normal to the hyperbola equation x=2secO,y=3un® atthe point where © == has _—_ equation ola 2x 3 gt 9s xtViy=13. hhas equation hyperbola x=2sec®, y=4tan@ at the point where o=-5 has equation 4 Y= 4416 = x-2y2y = 10V2 4) SD Solutions Series 3 Solution (a) The chord of contact of tangents from the point (5,4) to the ellipse 3 Xel 10 has equation 22 44¥ = 1 =» 5x+6y = 15. 15° 10 (b) 3x" +4y? = 485 3 4) to the ellipse —: (64) to the ellipse 6 seth tion = iy =| has equation += =) 9x+8y = 24 (©) The chord of contact of tangents from the point (1,2) to the hyperbola xy x_2y . ~~ =1 has equation =— cm as equation 2— = 2x-3y = 1. The chord of contact of tangents from the point (d) 9x? -2y _. (1,2) to the hyperbola S 4 Solution The normal to the ellipse =1 at the point P(acos®,bsin®) has equation ax by cos sin® ~b*. Point X has coordinates ( (7x9 - 16)(x, +8) = 0 TSD Solutions Series from the point (1,3) to the hyperbola “=~ -T= = are 16 y =—4x+7, with point of contact {2 y =2x+1, with point of contact P(-8,-15) 11 Solution Let the tangent at Po omeet x ain Q, R respectively. Let QR meet the y- axis in C. Tangent PR has equation xeos@ , ysin® _ a = e056) Hence @ has coordinates (Me) sin® and R has coordinates (-« sases®)) sin® 7 @ 2 2 Gradient QS x gradient Rs = PO=<080)_,_b(0* e080) ____ desos?@ a(l=e)sin6 —a(l+e)sinda*(1-e) sin? @ Then 6? = a?(1—e*) => gradient QS x gradient RS QSLRS. — Similarly, replacing e by -e, QS’ L RS’. Hence QR subtends angles of 90° at each of $ and S’, and Q, S, R, S’ are concyclic, with OR the diameter of the circle through the points. The y-axis is the perpendicular bisector of the chord SS’, hence the centre of this circle is the point C where the diameter QR meets the y-axis. SD Solutions Series 2B If (22) lies on the ellipse a y +y? =I, then QR has equation _ 3 Aine and meets the y-axis in C} 0,—= |. Also b’ = a?(1~e*) gives e? =—, and S has * 4 a) : coordinates (22,0). Hence cs -B and the circle through Q, S, R, S’ has 2 ( =) B equation x7+[y-—4.] -2 22) 8 ‘SD Solutions Series 24 Exercise 3.4 1 Solution (a) For the hyperbola xy=8 we have c? =8=>c=2V2. Hence the hyperbola xy = 8 has eccentricity = V2, foci S(cV2,ev2) = S(4,4) and S"(-eV2.-c¥2) = S(-4-4), directrives| xtyatev2 > xty=i4, asymptotes x=0 and y=0. (b) For the hyperbola xy=16 we have c?=16=c=4. Hence the hyperbola xy = 16 has eccentricity foci S(ev2,cV2) = 84v2,4v2) and S"(-c2,-cV2) = S(-4V2,-4V2), directrices x+y=tev2 = x+y=44V2, asymptotes x=0 and y=0 2 Solution (a) For the hyperbola xy = m S44 $(-4V2,-4V2) xty xhy=-4 we have c* =4 => c=2. Hence the hyperbola xy =4 ‘7SD Solutions Series c has parametric equations x= ct, y=—=> x= 21, y= t (b) For the hyperbola xy=25 we have c? =25=>c=5. Hence the hyperbola ay = 25 has parametric equations x= ct, y= £— x=5t, y 7 3 Solution : 4 (a) The hyperbola x= 41, y 4 has Cartesian equation xy = 41-5 = 2y=16. (b) The hyperbola x = 3, y = has Cartesian equation xy = 3t- 2 Sys 4 Solution (a) For the hyperbola xy=8 we have c* =8. Hence the tangent to the hyperbola xy=8 at the point P(x,,y,) = P(4,2) has equation xy, + yx, =2c> => x+2y=8 and the normal has equation xx, — yy, =x} — y? =» 2x~y=6. ql 1 y (b) For the hyperbola xy =12 we have c* =12. Hence the tangent to the hyperbola xy=12 atthe ~—point P(x,,9) = P(-3,-4) has equation ay, + yx, $20? = 4xt3y=-24 and the normal has ‘equation xy - yy, = x2 -yP S3x-4y=7. (©) For the hyperbola x= nya? we have c=2. Hence the tangent to the hyperbola x=2t,y=— at the point where 4 has equation t xtty=2er=pxtl6y=16 and the ~— normal ~—shas_—— equation ={e-F)s21-29=255, 3 we have ©=3. Hence the tangent to the (@) For the hyperbola x=%, y=2 1 has equation =3,y=2 at the point where + t hyperbola xt? y=2ct xt y=-6 and the normal has equation SD Solutions Series 26 § Solution (a) For the hyperbola xy=10 we have c?=10. Hence the chord of contact of tangents from the point T(x,,yo)=7(2,1) to the hyperbola xy=10 has equation 299 + Yq = 2c? =o x+2y = 20 (b) For the hyperbola xy =6 we have c’ = 6. Hence the chord of contact of tangents from the point T(x,.¥))=T(-2) to the hyperbola xy=6 has equation Yo + Xp = 2c? = 2x-y=-12. 6 Solution (a) The hyperbola xy=c? has parametric equations x=cr and y=. Hence Lif y=mex-+k is a tangent to the hyperbola at fon}, then 8 Pp. m= ap = mp? +1=0 dx a P lies on y=mx+k => mep-= > (2) (b) (1) x £4 2) = Imep =-k > ep P 2 - Wx£-2)=2=%5£=F P p 2 : kk Therefore the point of contact of the tangent y=me+k is {EE}. Now m tangents from the point (—1,-3) to the hyperbola xy =4 have equations of the form 7SD Solutions Series 2 y+3=m(x+l), that is, y=mux-+(m—3). Hence Amc? + k* = 0 => 16m + (m—3)? =0=9 m? +10m+9 =0= (m+ 1)(m+9) =0. kk LF} P-2,-2), EA) r2-a (4-4) Hence the tangents from the point (—1,-3) to the hyperbola xy = 4 are =-x~4, with point of contact P(-2,-2) and ) y=~9x~12, with point of contact A- 7 Solution The chord of contact of tangents from the point (—1,~3) to the hyperbola xy =4 has equation 3x+ y =-8. Let T(xp,Y) be a point of contact. Then T lies on the chord = 3x, +9 =-8, T lies onthe hyperbola => xy =4. Hence x, (-8—3x,) =4 => 3x3 +8xy +4 =0 => (3x+2)(x+2)=0 2 Fs Ye B= 3a = 6 or 2, ¥ 3x9 Equation of tangent at the point 7(x9,)9) i8 XY + YX = 2c”. Therefore the tangents from the point (~1,~3) to the hyperbola xy =4 are x—4, with point of contact P(-2,-2) and 9x—12., with point of contact A(- 3 6) f 8 Solution 1 The gradient of PR is —, the gradient of OR is p-r) pr eq-r) ar 1 Therefore PRLQR => gradient PR x gradient QR = -1 => par Pg ‘SD Solutions Series 28 The normal at the point Hone) has gradient r?, the gradient of PQ is Ge) Since 7? p-q) Pg Pg then gradient of the normal at R equals to gradient of PQ. Thus the normal at the point R is parallel to the chord PQ. 9 Solution ‘The tangent to the hyperbola xy=c? at the 2et. point afc.) has equation x +0? Hence the point T has coordinates et tet lee Tee == } The normal tthe hyperbola xy=c? at the point A«.2) 7 has equation noted . 1 rel nat) ae) +01? => OP = PN lee t 10 Solution TSD Solutions Series 29 Since R(x. q) lies on the tangent at the ; c 2 point Flep=], then x5 + p*ys =2ep. Since R(xo.¥9) lies on the tangent at the point oe), then x5 +9?y) =2cq. q. Thus x =222 and y,=—&. Then pt+q ptq OR has equation y=22. a The point M(x,,y,) lies on OR. Therefore Xo y, =. Since PQ is the chord of contact of tangents from the point R(x, yp), then Pq PQ has equation xy + yx =2c?. M(x,,y,) lies on PQ. Hence + y, =c2*4 Pq Pq Thus y, = i(é+4) and x, = +(cp+cq). Therefore M is the midpoint of PQ. PG. 11 Solution Let R has coordinates (9,y9). PQ is the chord of contact of tangents from R to the hyperbola xy=9. Hence PQ has equation xyy + yx, =18. Then (6,2) lies on PQ. ‘Therefore x +3yy =9. Thus the locus of R has equation x+3y=9. 12 Solution Let R has coordinates (x,y). PQ is the chord of contact of tangents from R to the hyperbola xy = c?. Hence PQ has equation xy + yxq = 2c?. Then (a,0) lies on PQ. ‘ 2c* ‘Therefore ayy =2c*. Thus the locus of R has equation y = ——— a ‘TSD Solutions Series Diagnostic test 3 1 Solution For the ellipse we have a=2,b=V3>b (44,0), a directrices: x=45— x @ asymptotes: y 3 Solution If P lies on the ellipse =I with the foci S(ae0) and S’(—ae,0), then ee ‘TSD Solutions Series 31 2 PS + PS’ =2a. For the ellipse i 2 a 1 we have a=3. Hence if PS =2, then PS'=6-2=4 4 Solution Since foci of a hyperbola are on the x-axes, then the equation of the hyperbola is 2 yt e are (4ae,0). Therefore the distance between the foci is 2ae=16. The equations of =1. Thus we need to find the parameters @ and b. Coordinates of the foci the directrices are x=". Hence the distance between the directrices is 2 e e a Thus we have two equations ae =8 and “= 2. From the first equation we get e e Substituting the expression for the e to the second equation we obtain a? = 8 Therefore a=4 and e= 2. Then b? =a°(e? ~1) = 16-(4-1)=48. Hence the 4 ay Cartesian equation of the hyperbola is = ae § Solution y “Tat Hence a=3 and b=2. (a) Cartesian equation of the ellipse is 9 Therefore the ellipse has parametric equations x=3cos® and y=2sin0, —n ZPs1_ at the point (2,1) has equation 5 Ra Lataoxs2y=4 The normal to the ellipse at z at the point (2,1) has equatior BBW ag-2-02e-y23 1 (b) The tangent to the ellipse x=4cos0, y=2sin@ at the point where @ “5 has 5 x xcos= — ysin- equation — = 1=x42V3y=8. The normal to the ellipse x=4c0s8,y=2sin@ atthe point where & has equation 16-4 6x- Vy =9 ze cos sin 3 y (6) The tangent to the hyperbola =—3-=1 at the point (4,3) has equation 4x_3y eae <2 =1=53x~-y=9. The normal to the hyperbola ~~ =} It aT y vyperbola “—-—2—= 1 at the poin (43) has equation me 2 2427 => xt3y=13. (d) The tangent to the hyperbola 3sec0, y= 6tan@ at the point where 0-5 has xsec= ytan™ 6 6 3 =34x-y=6V3. The normal to the hyperbola equation TSD Solutions Series =3secO, y=6tan@ atthe ~— point where «@ fas equation +36 x+4y=10V3 10 Solution (a) The chord of contact of tangents from the point (4,3) to the ellipse > 4x 3: has equation —* pals xt3y=2 (b) The chord of contact of tangents from the point (2,1) to the hyperbola 11 Solution ‘The tangent to the ellipse 1 at the point P(acos®,bsin@) has equation 22080 yin? » Point X has coordinates (asec8,0) and point Y has @ coordinates (0,bcosec®). Hence PX? = (acos@—asec6)? +b? sin? @ = a’ sin? @tan? 6+57 sin’ 6, PY? =a’ cos? 6+ (bsin@ — bcosec 6)” = a* cos” @+b? cos? Bcot? 6. PX _ Ytan’ 6(a’ sin? @ +5? cos? 8) Therefore —— = = =tan?6. If P is an extremity of a PY eot® a" sin® 6 +5" cos? 8) latus rectum, then acos® = -2e. Thus cos0= +e. Hence 2% = 12 Solution The normal to the hyperbola =; a 1 at the point P(asec®,b tan) has equation 2 ax 2 +] =a" +b*. Point X has coordinates sec tan® 0 and point ¥ has ‘SD Solutions Series 35 sec? 0+b* tan? 0= 2 (b%50c? 04a? tan? ) a 7 ‘ 2 ) tan? 0 = a? sec? 0+ tan? ae sec* @+a” tan’) > Wr sect ea Pb sechOra7tan?® yo Therefore PX = 2 Be PY Sor sect Ora? tan? 4 13 Solution For the hyperbola xy =18 we have ch =18=>c=3V2. Hence the hyperbola xy = 18 has eccentricity e = 2, foci S(ev2,cv2) = $(6,6) and S%-eV2.-cV2) = S(-6,-6), directrices x+ystev2 = x+y=36, asymptotes x=0 and y=0 14 Solution (a) For the hyperbola xy=9 we have c? = 9=> c=3. Hence the hyperbola xy=9 has parametric equations x = ct, y= . sx=3ty (b) The hyperbola x =5t, y = 2 has Cartesian equation xy = 5 2 =xy=25. ‘SD Solutions Series 36 15 Solution (a) For the hyperbola xy=6 we have c? =6. Hence the tangent to the hyperbola xy =6 at the point P(x,,y,) = P(3,2) has equation xy, + yx, = 2c? =» 2x+3y=12 and the normal has equation xx, ~ yy, = 47 —y} = 3x-2y=5 (b) For the hyperbola x=4r, y we have c=4. Hence the tangent to the 4 hyperbola. x=4r, at the point where 1=2 has equation xty=2ct=x+4y=16 and the ~—pormal,~—has_——equation wobede )=+4x-y=30 1 7 (©) For the hyperbola xy =4 we have c’ =4. Hence the chord of contact of tangents from the point T(xp.y))=T(2,-1) to the hyperbola x 4 has equation 0p +My #207 = +2 =8 16 Solution The tangent to the hyperbola xy=c? at the point acs } has equation x+0y=2ct. Hence the point X has coordinates (2ct,0) and the point ¥ has es 2c) coordinates (o +) t and (b) The area of AYOX is 4-0X OY = a ier independent of 7SD Solutions Series 37 Further questions 3 1 Solution ‘The tangent to the ellipse xcos8 | ysind a é (a) The point A has Ay coordinates (asec 8,0) a and the point X has coordinates (acos®,0). Hence a OX OA =acos0-asec@=a? (b) The point B has coordinates (O,bcosec6) and the point Y has coordinates (0,bsin@). Hence OY-OB = bsin8-beosec@ =b*. 8 — 1| (©) Since $ has coordinates (2e,0), then sT=—-EES°= Since 5 has cos? 8 coordinates (-ae,0), then “Therefore oo ST-S'T’ = 1225 8_ But for the ellipse b? = a2(1—e*) =3 e? cos? @ | sin?” 78D Solutions Series 2 Solution ‘The tangent to the hyperbola xsec@_ytan® a b (a) The point A has coordinates (acos6,0) and the point X has coordinates (asec®,0). Hence OX -OA = asec8-acos8 (b) The point B has coordinates (0,-bcot®) and the point Y has coordinates (0,btan). Hence OY- OB = btan8-beor 8 =b* (©) Since’ $ has coordinates (ae), then sr-=— sl +=: Since S’ has coordinates (-ae,0), then sec?@ tan?@ But for the hyperbola ‘TSD Solutions Series 39 3 Solution 2 The normal to the ellipse +25 =1 ab at the point P(acos®,bsin®) has b?. The coordinates -b ON . But for the ellipse b* =a(1-e?) => . Thus OG = e’ON (b) Since the focus $ has coordinates (ae,0), then SG=|ae- cos6| = ae(1- ecos®) and SP = y(ae—acos0)* +b? sin? 6 = ay(e-cos8)’ + (1—e*) sin? 6 = av 1—2ec0s0 +e? cos* @ = a(1—ecos6) Hence SG = eSP.. Since the focus S’ has coordinates (-ae,0), cud = ae(1+ecos6) and S‘P = (—ae—acos®)* +b? sin? @ = ay(e+cos®)* +(1-e*)sin? @ = av/1+2e.cos6 +e" cos? 0 = a(1+ ecos8) Hence S’G = eS’P 7SD Solutions Series 40 4 Solution The normal to the hyperbola P(asec6,btan6) at the point P(asec®,btan6) has equation +b? . . The point G has coordinates (: 0] a : ae (a) The point NV has coordinates (asec0,0). Therefore 0G = 2 2 ON . But for 14g? the hyperbola 8 = a*(e? 1) 3.2% = 6+. thus OG = ON. @ (b) Since the focus S has coordinates (ae,0), then SG = a and SP = \(ae—asec®)’ +b" tan’ 6 = ay(e~sec®)? +(e? — 1) tan? = avi 2esecO +e” sec? = all —esec | Jae sec 6) = ae|t—esec 0] Hence SG = eSP.. Since the focus S’ has coordinates (~ae,0), lay then so-boet oe st] adres @ and S’P = .j(-ae—asec)? +b? tan? @ = ay(e + sec)” +(e? —I)tan?@ = ayl+2esecO +e? sec? 0 = all +esec | Hence S’G=eS’P, 7SD Solutions Series a 5 Solution The tangent to the ellipse 1 at the point P(acos®,bsin®) has equation xo0s0 , ysin® Hence the a point @ has coordinates (0,bcosec 8). Thus the gradient of beosec® ae QS’ is The gradient of =1 at the point P(acos®,bsin@) is 2508 beos8 the normal to the ellipse P lies at an extremity of a latus rectum through the focus S(ae,0),, then cos@ =e and =>. Therefore the gradient of Qs’ is &.2 a ae sind = and the gradient of bea the normal at P is © - Hence the normal at P is parallel to OS’ bea e 6 Solution ‘The tangent to the ellipse at the point P(acos6,bsin®) has equation xcos@ | ysin® crooseT™ yan) a > =1. Hence the point R has coordinates (o.Micsae)) Thus the : sin@ gradient of on is PUTS) aye gradient of | A’P is asind bsin® bsinO(-cos®) _ bsinO(I~-cos®) _ b(1- 059) ay erefore OR a(cos8+1) a(cos@+1)(I-cos®) a1 —cos* 8) asin® ‘SD Solutions Series 2 is parallel to A’P 7 Solution The tangent to the ellipse x?+2y7=19 at the point P(xy,y9) has equation 7 2X9 +2yVo = 19. If this tangent is parallel to x+6y=5, then ae 6 = yy = 3Xp- o Since the point P(xy,y) lies on the ellipse, then xj +2y;=19. Therefore x} +2-9x2 = 19 => x2 = 1. Hence the tangents to the ellipse x? + 2y? =19 are x+6y=19, with point of contact P(1,3) and x+6y= 19, with point of contact P(-1,~3) 8 Solution The tangent to the hyperbola 2x? ~3y? a the point P(x.) has equation 2. 2xx, ~3y% =5- If this tangent is parallel to &x=9y, then 22-85, 3, 3y, 9 Since the point P(x,,yo) lies on the hyperbola, then 2x; -3y2 =5. Therefore a 9 2x5 - 3, 16 8x—9y =5, with point of contact P(4,3) and xg =5=> x} = 16. Hence the tangents to the hyperbola 2x* -3y? = 5 are 8x—9y =~5, with point of contact P(-4,-3). 9 Solution The tangent to the hyperbola x? ~ y* at the point P(t», yo) has equation ; 4 2X» — Wo = 7. If this tangent is parallel to 3y = 4x, then 2% za Yo the point P(x,.¥9) lies on the hyperbola, then x3 —-yg = cae 2 2 x [eX = 72 45 = 16. Hence the tangents to the hyperbola x 4x—3y=7, with point of contact P(4,3) and 4x~3y=~7, with point of contact P(-4,-3) ‘SD Solutions Series 43 10 Solution ‘The tangent to the ellipse 8x*+3y?=35 at the point P(xo,y,) has equation 8xx,+3yy)=35. The point (23) lies on this tangent. So 7 10x, +15y =35= yo = ~ 2a. Since the point P(x,,¥9) lies on the ellipse, then 8x) +3yp = 35. 7.2.) ‘Therefore ag+3.(2-255} = 35 => 28xj — 28x) —56= 0 = (x) -2)(xy +1) =0 i) are 4 Hence the tangents to the ellipse 8x? +3y* = 35 from the point G 16x-+3y =35, with point of contact P(2,1) and ~8x+9y '5,, with point of contact P(-1,3). 11 Solution ‘The tangent to the hyperbola x’-9y?=9 at the point P(x,,y,) has equation X%y—9yyp=9. The point (3,2) lies on this tangent. So 3x5 - 18 yp =9 => x) = 3+ 6yp. Since the point P(x,,¥9) lies on the hyperbola, then . Therefore (3+6yo)? —9yg =9 => 3y3 +4yy =0=> yo(3y, +4) = x2-9y2 = ‘Hence the tangents to the hyperbola x? - 9y? = 9 from the point (3,2) are x=3, with point of contact P(3,0) and -5x+12y=9, with point of contact ct ‘7SD Solutions Series 44 12 Solution The chord PQ of the ellipse y4 _P(acos®,bsind) = 1 has equation |A(a,0) 28), 2222) wa 28 fe. where P, Q have parameters 0, ‘Qacosd,bsing) . The chord PQ cuts the x-axis at point T(1,0). So t= acod 28 Jae 222) bsin® eas=na Zon and the gradient of AQ is o(unSin$ (1-tan tn ee: 2 The gradlent of AP is b Sine __beo.® i the chord PQ subtends a right angle at the point A, then a(cosp—=1) a2 Bp Therefore “cot cot a2 e 1 => tan=tan 2 « But for the ellipse 5? = a?(1-e”). Thus a+b? gradient AP x gradient AQ = ay 2 2\' Hence ob f-4) ze So PQ passes through a fixed point 10] on the x-axis, +e 13 Solution ae e The normal to the ellipse at the point P(acos®,bsin8) has by a equation ——. -b?. The eee c088 sind point G has coordinates ( cos 8, ) Therefore a ‘TSD Solutions Series 4S PG? (0-2 } cos? 6 +b? sin? @ = 7 (b? cos" 8 +a? sin® 8). a a But for the ellipse 6? = a?(1—e?). Hence PG? =a"(i~e?)(1—e? cos? 0) From the other side PS? =a?(e—cosb)" +b? sin? = a*(1—2ecos0 +e? cos? @) = a?(1—ecos8), PS’? = a*(e+cos8)* +b? sin? @ = a?(1+2ec0s0 +e? cos? 0) = a*{1 + ec0s8)? ‘Thus PG? = (1—e*)-a(1—ecos®)-a(1+ecos®) = (1?) PS: PS’. 14 Solution ‘The tangent to the hyperbola xy (aay P(asec®,btand) has =1 at the point equation xsecO_ ytan@ a b The point T has coordinates (0,-beot 8). The normal to the hyperbola at the point by 245, The point G has coordinates P(asecO,btan6) has equation —= see tend 2 yp ley 4 (02 1 sm). So gradient SG x gradient ST = 27% tang. 258 Since for é ~ae 2 atte? . ae the hyperbola b* = a? (e? ~1) then gradient SG x gradient S7 =~" >? =—1, Thus ae SGALST and consequently GT subtends a right angle at focus S. Similarly es a gradient Gx gradient T= "= tang een ene 23 ‘Thus ae ae ae’ S’GLS’T and consequently GT subtends a right angle at focus S’. Therefore 5G, sT are concyclic with GT the diameter of the circle through the points. ‘78D Solutions Series 46 15 Solution ‘The normal to the hyperbola ~~ Sat atthe point P(asec®,btan®) has equation ? +b®. So the point N has coor Gee ws (2 # see0). Since the @ 2) a asymptotes have equations y=+—x, then the point Q has coordinates (asec0,tbsec®) Thus the_~— gradient’ «—of on is {(a mbsecO {( — a}=8] ™ Therefore @N is perpendicular to the a asymptote. 16 Solution 5 b Let @ denotes the smallest angle from positive x-axis to the asymptote y=— a b 2itan Since tan ==, then wna =| ane | = a tan? @ B( 17 Solution The normal to the hyperbola xy at the point feu$} has equation ? wtefe-4) Let the point Q, R have coordinates (x,y) and (x,,94) respectively. Since Q, R lie on the hypesbola x? — ) then OF — 4D) Of — 92) = 09 Om Va #4) = (91-2 + Ye) w The points Q, R lie on the normal to the hyperbola. Therefore ‘TSD Solutions Series 47 (2) + HG $x,)- 2 = @ Substituting (2) into (1), we obtain tm = POV +I) (4) 2, L ‘Then B.A) = PO.+ ¥)- FO tn) = © Using (5) we get from (4) x, +x, =2ct 6 Thus, according to (5) and (6), ihe midpoint of QR has coordinates («<). Hence A the point Aa) is the midpoint of QR. 18 Solution The normal to the hyperbola xy=c? at the point end) has equation 7 ={e-3). The point da) lies on the normal. Hence 1 r q e_f{p_4 2 ; teq—En di -4)5 (0 {iso Since QP, then q #1. Therefore 4 q 1 q “+ and Q has coordinates (-S-<") ‘The point (er) lies on the circle r r on PQ as diameter. Hence gradient RP x gradient RQ = 1. But gradient of RP is ‘. 12-1 and gradient of RQ is dt-1).1 _--+. rus t)ar-) rt rq) cr-4) 79 +L. since = ‘Therefore r=—t, because tq 7SD Solutions Series 48 R# P. Sothe point R has coordinates ( ct 19 Solution If M(a.,) is the midpoint of AP, then x= (sec@+1) and y =F tand. Therefore (2x-a)? -(2y)? = a*(sec? @— tan? 6) =a?. Hence the locus of M is hyperbola (2x-a) ~(2y)? 20 Solution The normal to the hyperbola xy=c? at the point («.<) has equation t ): The 2] lies on the normal. Hence q =0. Since Q# P, then q #1. Therefore a=-$ and Q has coordinates ( Se), If M(x,y) is the midpoint of PQ, then e acl x Surg={e 7) aM and g(5-4 ; ) We obtain from (7),(8) that 2 = 7 . Substituting this formula for t? into (7), we get = iin Saar eet ay 2|-¥ 7 4y xty’ ‘Therefore the locus of M has equation 4x°y? +¢?(x? — y?)? ‘7SD Solutions Series 7SD Solutions Series Worked Solutions to Popular Mathematics Texts Suggested Worked Solutions to “4 Unit Mathematics” ( Text book for the NSW HSC by D. Arnold and G. Arnold ) Chapter 4 Polynomials INOS g , COFFS HARBOUR Seon QR cow MT Solutions prepared by: Michael M. Yastreboff and Dr Victor V. Zalipaev 7SD Solutions Series Worked Solutions to Popular Mathematics Texts EX page Exercise 4.1 1 Exercise 4.2 9 Exercise 4.3 13 Exercise 4.4 . 20 Exercise 4.5 27 Diagnostic test 4 31 Further Questions 4 39 Solutions are to “4 Unit Mathematics” [ by D. Amold and G. Amold (1993), ISBN 0 340 54335 3 ] Created and Distributed by: 7SD (Information Services) ABN: 13009821 © 780 1987 ‘The 7SD team welcomes all feedback. Correspondence should be addressed to: 7SD atin: Michael Yastreboff PO Box 123 Kensington NSW 2033 Exercise 4.1 1 Solution (a) (i) In order to solve the polynomial equation P(x)=x*—5x?+4=0, denote x? as t then 1? -5t+4=0=9 4 =4, 1 =1. Hence 1? —St+4=(¢—4)(t—1) and thus x8 5x7 44 = (x? —4Y(x?=1) = (x—2)(x + 2)(x— I(x +1) are inreducible factors of P(x) over Q. Each linear factor gives rise to a zero of P(x). Hence the zeros of P(x) are +1, 2. All these zeros are rational. i, ii) Over R and C this polynomial has the same zeros. (b) Denoting in P(x) = x*-3x?+2=0, x? as ¢ we obtain 17—3t+2=(t-2\(~1), and thus x*—3x? +2 = (x? -2\(x? —1)=(x?-2)(x~ (r+ I) are irreducible factors of P(x) over Q. Hence the zeros of P(x) over Q are +1. (ii, iii) Inreducible factors of P(x) over R and also C are P(x) = (x V2 (x tV2)@ I(x 41). Hence the zeros of P(x) over R and C are +1, +2. (c) Gi ii) Irreducible factors of P(x) over Q and R are P(x)= (2? ~ DG? +4) = (x= +? +4). Hence the zeros of P(x) over Q and R are +1. Gil) PX) = (x (x4 (x~ 20x +2) Hence the zeros of P(x) over C are +1, +2i. 2 Solution (a) @ P(x)= x4 5x? +6 = (x? -2)(x? -3), and these factors are irreducible over Q. Hence P(x)=0 has no roots over Q. Gi, iil) P(x) = —V2)(x + V2 (x — V3)(x-+-V3). Hence the roots of P(x)=0 over R and also C are +y2, +3. TSD Solutions Series (b) (i) PQ)= 2 9 = (x2-2)(x? +1) = the equation P(x) =0 has no roots over Q. (i) P(x) =(x-V2x + V2)(2? +1) = the roots over Rare +¥2. (Gilt) P(x) = (x— V2) + V2) D(x + 1) = the roots over C are (©) (i, ii) P(x) =(x? + 1)(2? +4) and cannot be factored further over Q and R = P(x)=0 has no roots over Q and R (iii) Irreducible factors of P(x) over C are P(x) =(x~i)(x + i)(x-2i)(x+24) Hence the roots of P(x)=0 over C are ti, +21. 3 Solution (a) x tl quotient x= O« remainder 9 (8 -x? +x-1)=(x-IQ? +1). Also P(x)= 8 =x? +x-1= PUL (b) x? +(i-1)x-i quotient G@-)x+ ox =I 2 +(+ix vir-1 crix=1 ‘SD Solutions Series 0 remainder exritex (x i){x? + (7-1) x- i}, Also PC) P+i-1=0. 4 Solution (a) xtl)x3 3x? 442-2 etx? quotient 4x? +4x-2 Wax? - 4x 8x-2 Bx+8 10 € remainder => (9-32? +4x—2)= (xt 1)(x? 4x + 8)-10. Also P(x) = x9 3x? +4x-2= P(-I)= 3-4-2=-10. (b) (3+ i)x-+(3i+3) quotient 3x? +4x-2 (3-ix?+ 4x-2 (3-7 +3i+ x (3i+3)x-2 (343) x43i-3 1-3i < remainder = (x9 = 3x7 44-2) = (x4 x? - Bt ix + Bi+3)}+(1-3i). Also P(x) = x? ~3x? 44x~2= P(-i)=i+3-4i-2= 1-31. 5 Solution (a) @ PQx)= x) +x? -3x-3= x(x +1) Hx +1) = (x+1)(?—3) are irreducible factors over Q. ‘SD Solutions Series, (ii, iii) Imeducible factors of P(x) over R and also C are P(x) = (2+ (x-V3)(x+V3). (b) Git) P(x) = 33 2x? 4x8 = 22(x-2) + 44-2) = (2 2)(x? +4) are irreducible factors over Q and also R. (iii) Irreducible factors of P(x) over C are P(x) = (x—2)(x—2i(x+2i) 6 Solution (a) (i) The only possible rational zeros of P(x) are +1,+2,+4,+8 (integer divisors of the constant term 8). But of these, only 1 and 4 satisfy P(x) =0. Hence (x~1) and (x—4) are factors of P(x). By polynomial division’ P(x) by (x= (e+ 4)= 2? +3x—4 we obtain P(x) =(x-D(x+4)(x? —2) and these are irreducible factors over Q. (ii i) P(e) = (eI + 4)(x? = 2) = (x Dx + 4)(x—V2)(x +-V2). (>) (iii) The integer divisors of the constant term ~6 are +1, £2, 3,46. Of these only -2 and 3 satisfy P(x)=0. Polynomial division P(x) by (x+2)(x~3) yields P(x)= x4 = x3 -5x? —x-6=(x+2)(x—3)(x? +1), and these are irreducible factors over Q and R. (iil) PO) = (x + 2) +3)(x? +1) = (x +2) + 3)(X-D(x+). 7 Solution (a) According to the condition of the problem and factor theorem factors of P(x) are (x~5) and (x +2). Hence P(x) = (x—5)(x+2)? = (x -5)(x? +4x44) = x9 —x? - 16x20 (b) Factors of P(x) are (x+1) and (x-3)*. Hence P(x) =(x+1)(x—3) =(x+1)(x? - 9x? +27x-27) = x4 8x9 +18x7-27, ‘SD Solutions Series 8 Solution P(x) = x3 -3x? +4-=> P(x) = 3x" - 6x =9 P(0)=0 and P'(2)=0, but P(0) #0, P(2)=0. Hence 2 is a multiple zero of P(x). As P”(2)#0, its multiplicity is two. 9 Solution Investigate rational roots of P(x)=0. Among integer divisors of the constant term ay = 2 of P(x) only ~i and 2 satisfy P(x)=0. P'(x)= 4x +3x? -6x~5 => P(2) #0 and hence 2 is a single zero. P“(-1)= 0, P’(x) =12x" + 6x—6=> P”(-1)=0. PO(x) = 24x +6 => PO (-1) #0. Hence —1 is a root of multiplicity 3 of P(x)=0. As P(x) is a polynomial of degree 4, P(x)=0 has no other roots exept for 2 and -1. 16 Solution P(x) = 4x3 +12x? 15x44, P(x) = 12x? +24x-15, P'(x)= 24424. => P'(J/2)=0, P(-5/2)=0. But P(1/2)= ), P(-5/2) #0=> 1/2 is a multiple zero. As P"(1/2)#0,1/2 is double zero, and (2x~1)? is a factor of P(x). By polynomial division P(x) =(x-+4)(2x—1)*. These are irreducible factors over R. 4 isa single zero, 1/2 is a double zero. 41. Solution P(x) = x4 —3x3 — 6x? +28x-24, P(x) =4x?—9x7 124428, P"(x)=12x? ~18x-12, P(x) = 24x18 => P"(2)=0, P’(2)=0, P(2)=0, P(2) #0. TSD Solutions Series Hence 2 isa triple zero of P(x) and P(x) =(x-2)°(x+k) for some constant k , as P(x) is a monic polynomial of degree 4. Then P(0)=-24= k=3 and P(x) =(x-2)'(x+3). So -3 is a triple zero of P(x) 12 Solution P(x)=x3-3x7-9x +0, P(x) = 3x? -6x-9, PM(x)= 6x6. = PI) =0, P*(-1) #0, P’G)=0, P”(3) #0. Hence both —1 and 3 can be a double zero of P(x) Let ~1 be a double zero of P(x).=> P(x) =(x+1)*(x+k) for some constant k, as P(x) is a monic polynomial of degree 3 P(0)=c=> k=c. P(-1) =O c=~S> P(x) =(xt1)*(x-5), Let 3 be a double zero of P(x).=> P(x) =(x-3)"(x+) for some constant 1, P(3)=0=9¢=27. PO) =e l= 5 = 3.9 P(x)=(x~3)(x4+3). 13 Solution P(x)= x4 +2x°-12x7- 40x40, Pi(x)= 4x3 +6x7—24x-40, P(x) = 12x? +12x-24, P(x) = 24x +12.=> P”(1)=0, P"(~2)= 0. P’(1) #0, P’(-2) = 0, P(-2) #0. Hence -2 isa triple zero of P(x) and P(x)=(x+2)°(x +k) for some constant k, as P(x) is a monic polynomial of degree 4. Then P(-2)=0= ¢=-32. P(0)=c=> k=—=—4 and P(x)=(x+2)(x~4), 14 Solution Podsaxdtbr +d, ‘SD Solutions Series P(x) =3ax? +2bx, "(x)= 6ax+2b. => P'(0)=0, 2) 0. Hence both 0 and —2b/ (3a) can be adouble root of P(x)=0. Let Obe a double root. Hence P(0)=0=> d = 0=> if 27a7d +4b* =0, then b=0=9 P(x)=ax*and 3is a triple root. Thus if Ois a double root, then 21a*d+4b #0. Let -2b/ (3a) be a double root of P(x) =0. Hence 3 2 A261) =0= a 22) +0(2) +d=0=210'd+4b' =0. 15 Solution P(x)saxtex+d, P(x)=3ar?+0, P(x) =6ax. =r(- =o, wee e=+1.Hence 3a ec => = 4c" + 27d” 16 Solution 2 7 If P()=1-x—- 2-1 +(-1" 2, then 2 n! yet P(x) =-143-L 4-1)" eee rear => P(x) P(x) = 2px)", nl (1)Suppose a is a multiple zero of P(x), then P(ct)= P’(a.)=0, and ‘7SD Solutions Series P(a)- P(a)= , using (1)=> a = 0. But P(0)#0. Hence P(x) has no multiple zero. 17 Solution But of these, only (4) The only rational zeros of P(x) are £1, 3 satistes P(x)=0. Hence (2x~3)is a factor of P(x). By polynomial division, 2x3 3x? +2x—3=(2x—3)(x? +1), and these are irreducible factors over R. 2 But of these, only —1 2 2 P(x) =0. Hence (2x +1)is a factor of P(x). By polynomial division, isfies (b) The only rational zeros of P(x)are +1,+2, 2x3 +x? —4x—2 = (2x-+1)(x? — 2) = (2x + I(x-V2 (x +-V2), and these are irreducible factors over R. 18 Solution The only rational zeros of P(x)are 21,43, +4 But of these, only jan ~2 satisfy P(2)=0. Hence (21-1) and (2x +3) re factors of P(x). By polynomial division, 4x4 +8x +5x? + x-3=(2x—1)(2x +3)(x?+x+1), and these are irreducible factors over R, as (x” +x+1) = Ohas no roots over R and so cannot be factored further over R. ‘SD Solutions Series Exercise 4.2 1 Solution (a) x+i isa linear divisor. Hence we can use a remainder theorem, and the remainder is P(~i) = (iP +A(~s t= i-241=-14i. (b) P(x)=x? +2x? +1 and D(x) =x? +1 are polynomials over Q. By the division transformation, P(x) = D(x)S(x)+ R(x) where R(x) is a polynomial over Q, such that deg Re deg D =2. Thus P(x)=(x?+1)S(x)+ax+b, a,b rational, = P(i)=0+ai+b, and hence ait b=i +27 + -i=a=-1,b=~-1. Hence the remainder ax+b is -x-1. 2 Solution (a) x~2i is a linear divisor. Hence we can use a remainder theorem, and the remainder is P(2i) = (2i)° — 3(2i)* + 2(2i) - 1 = 321-484 41-1 = 49+ 367. (b) P(x)=x°—3x4+2x~1 and D(x)= x? +4 are polynomials over Q. Hence P(x) = D(x)S(x)+ R(x), where A(z) is rational and deg R< deg D =2. Thus P(x) = (x7 +4)S(x)+ax+b, a,b rational. = P(2i)=0+2ai+b , and hence 2ai+ b =(2i) -3(2i)4 + 2(2i)-1=32i- 48 + 4i-1=361~ 49 = a= 18, b=-49. Hence the remainder ax+b is 18x-49. 3 Solution By the division transformation, x*+ ax? + 2x = (x? +1)S(x)+2x+3. Substituting -a+2i=2i+3a5 SL) Solutions Series 10 4 Solution By the division transformation, x‘ tax? +3x-1=(x7 +4) G(x) +x45. Substituting x=2i, 16~8ai+6i-11=2i+5=>a=1/2. 5 Solution By the division transformation, x*+ ax? + 6x +2 = (x? +1)S(x)—x +1. Substituting i+, thatis -a+bi=-i-2 a= 6 Solution By the division transformation, x4 + ax? +b = (x? +4)S8(x)—x +13. Substituting x =2i, we obtain 16—8ai+ b= ~-2i+13, thatis —8ai+b= ~3a51/4,b=-3 7 Solution If =x+iy, then yr-x?42ni=4i=> wys2 Py =(x-y(xty)x?-y?=0 = eee if and only if y=xory=—x.Hence as seen *¥=2 from the graphs there are two points of intersection. 8 Solution Consider the polynomial A(z)= P,(z) which has more than n zeros. But deg (R~ P,) Sn. Itis possible only if R(z)— P,(z) =0 identically. Hence R(0)- FO) =0=9 by =e, A (0) Py (0) =0=9 = 4, ,RO(0)~ Pf(0) =0 b, =c 9 Solution P(x) has real coefficients, Hence P(i)=0=> P(—i)=0 and then (x-i)(x+i) =x? +1 isa factor of P(x). By the division transformation P(x) = x8 xP =x? px 2= (x? H1Y(x? + 2-2). PC) = (2? +1) = 1) +2). This ‘7SD Solutions Series u is the factorisation of P(x) into irreducible factors over R, and P(x) has zeros i,~2 and 1 over C. 10 Solution P(x) has real coefficients. Hence P(2—i)=0=> P(2+i)=0 and then [x-(@2-d}-[x- +d] =x? ~4x45 is a factor of P(x). By the division transformation P(x) = x4 5x3 + Tx? +3210 = (x? — 4x +5)(x? x —2).=9 P(x) = (x? 4x +5)(x+ 1x2) « This is the factorisation of P(x) into irreducible factors over R, and P(x) has zeros 2-i,2+i,-1 and2 over C. 11 Solution P(x) has real coefficients. Hence P(1+2i)=0=> P(1~2i)=0 and then [x-0+21]-[x-@-28]= x? -2x-+5 is a factor of P(x). By the division transformation P(x) = x*—2x° + 6x? —2x +5=(x7 -2x+5)(x7 +1). This is the factorisation of P(x) into irreducible factors over R, and P(x) has zeros (1+2i),(1-2i),-é and i over C. 12 Solution P(x) has real coefficients. Hence Ai) =0= P(-i and then (x- ix +i) =x? +1 is a factor of P(x). The rational zero of P(x) is p/q, where q is a divisor of the leading coefficient 1 and p is a divisor of the constant term 3. Hence P(x) has the form (x?-+1)(x—0t )(x—B ), where the rational zero ot takes one of the values +1, +2, or +3 (since P(x) is a monic polynomial of degree 4). Given that the constant term is 3=> 0. B=3, and hence the zeros of P(x) are i,~i,1 and 3 ori, J and -3. But the sum of the zeros is negative. Thus P(x) = (x? +1)(x4+1)( +3), and these factors are irreducible over R. ‘TSD Solutions Series 2 13 Solution xttax? +b. P(x) P(x) is an even monic polynomial of degree 4. Hence P(x) has real coefficients. Hence P(2i)=0-=9 P(~2i)=0 and then (x-2i)(x+2i) =x? +4 is a factor of P(x) => P(x) =(x? +4)(x? +c) The product of zeros of P(x) is -8. Hence 4c =-8 => c=—2, and P(x) = (x? +4)(x? =2)= (x? +4)(x— V2)(x+-V2). These are irreducible factors of P(x) over R, and P(x) has zeros ~2i, +2i,~V/2 and V2 over c 14 Solution Plxyaxt tax’ +bx? tex +4. P(V2)=0= (8+2b)+(2at+e)v2=0, a, b, cinterger = 8+2b=0 and 2a+c=0. Hence P(x)=x* tax? — 42? —2ax +4 = (x? -2)? + ax(x? = 2) = (x? ~ 2x? +ax-2). Thus x? +ax~-2=(x~o )(x+f) has a rational zero, which may be +1, +2 But @f =-2= a@=-1, 8 =2 or a=1, B =-2. The sum of zeros of P(x) is positive. ~1,B=2. Thus P(x) = (x? —2)(x + 1)(x-2) = (x-V2)(x+ V2)(x-+1)(x—2), and this is the Hence -J2+V2+a+B=a+B>0=> factorisation of P(x) into irreducible factors over R. ‘TSD Solutions Series 3 Exercise 4.3 1 Solution P(x)=x3+ax?+bx+c, since P(x) is the monic of degree three. If = 1,B=2,y=3 denote the zeros of P(x). then -Ya=-(1+2+3)=-6, Yap=2+3+6=11, c=-Lapy=-6 Hence P(x) = x? - 6x? +11x-6. 2 Solution P(x)=x* +ax3+bx? +ex+d, since P(x) is monic of degree four. , Y= land 8=3 denote che zeros of P(x), then ~S.a=-(-3-141+3)=0, Lap=3-3-9-1-34+3=-10, -LaBy=-G+9-9-3)= d=Sapys=9. Hence P(x) = x* 10x? +9. 3 Solution Let the roots of P(x} be a, a ‘Then product of roots is => B= 2 1 products taken two at a time is W2420=-Boa=3t, a a Sum of the roots is aehe2 = a=-16. Hence the roots are ah2 and the coefficient a is -16 4 Solution Let the roots of P(x) = x9-3x? -4x +a be 0, ~a,B. Then sum of the roots is B.=> 3. Sum of products taken two at a time is 0? +3030 =-c0? = 9? =-4 > = 2,2. ‘SD Solutions Series 4 Product of the roots is -2-2-3=-a=9 a=12. Hence the roots are -2, 2,3 and the coefficient 5 Solution 1 Let the roots of P(x)= x4 + pr>+gx? +rx+5 bea, q'PiB. Then =p 1 s=LaBy=a-2-B(-B)= Hence q=Lag= L L352 =1-p% +0-B—a-B+2-B-2-B-B?=1-Bi => Br =1+s. a -p=Za-a+l+p-prarta-p Hence 1 LaBy=a ipa + (-B)+a-B OL BP=-{art p= ~p(-s)= ps. 6 Solution ‘The sum of roots is BaYa=(anc)tat(ate)=3as aa, Pp 2 P 3 ~q)_ PT Gd a = 2 Hes 0= re) = #2} areieors a aeeeuenay aes 2q°-9pqr+27ps 7 Solution Let the roots be a—c,a,a+c. Then -22 =Sa=(a-d+a+(ate)=3a— a=—4 4 5 AaLepy=(a-ea(are)= (or e=~2 that gives the 1 same roots). Hence the roots are a~¢= and arena. ‘7SD Solutions Series 15 8 Solution Let the roots be b-c, b b+¢. Then 6= Da=(b-c)+b+(b+c)=3b=9 b=2. -10= Ya-B-y=(b-c) b (b+c)=2(4—c*) = ¢=3 (or c=—3 that gives the same values of the roots and so the same constant a). Hence the roots are b-c =-1,b=2,b+e=5 and a= Pa-B=-2-5+10=3 9 Solution ‘The product of the roots is -— = a-B-y=ac-a P Thesumotiherootsis == Fax a-ctarS=a(c+1+-) andthe product of ? c ¢ the roots taken two at atime is = Sya-P=a%e-ha? +. H(esi+4), P €, Hence —4. P 10 Solution Let the roots be a-¢, a. Then c Bayehyeacetne 1 Gedare crate af cvs c 3 =a=-2, crt a tg ata ite +4a0—c=-t c 4 4 (or c=—4 that gives the same roots). Hence the roots are a-c =}. 11 Solution Let the roots be b-c, b, 2. Then © ‘ISD Solutions Series ill 16 27=Da-P-y=be-b- 2=b > b=3, ¢ b 1 = = b+==3-+1 B=Lasb.cro+e {i +e}= and using quadratic formula c= 3 (or c= 1/3 that gives rise to the same values of the roots and so of the constant a). Hence the roots are b-c=9,6=3,2=1 and a=Sa-P=2749+3=39. © 12 Solution ‘ 3 2 (a) The values 20, 28 and 2y satisfy () -{3) ~{2)-2=0 and hence the required equation is x° + 6x? —8x—-16. (b) The values @—2,B-2 and y—2 satisfy (x +2)? +3(x+2)?-2(x+2)-2=0 and hence the required equation is x? +9x?+22x+14=0. fete Dre (¢) The values > satisfy () +2) ~{t}-2=0 and hence the required a'B'Y x x x. equation is 2x? +2x?-3x-1=0. 3 2 (d) The values a, B® and y? satisfy (x”) +3(x”?) -2x”?-2=0 Rearrangement gives x!(x—2)=2-3x. Squaring we obtain x(x~2)* =(2—3x)? and hence the required equation is x°+9x?+22x+14=0. 43 Solution (a) The values 2cr, 2B, 2 and 28 satisfy the equation 4 3 2 x x) _fx x : () + 3) 3) -3) +2=0, Hence the required equation is x4 +8x°-12x?-32x+32=0. ‘SD Solutions Series a (b) @=2,B-2, 9-2 and 5~2 satisfy (x+2)4 +4(x +2) (xt 2)? -4(x +2) +2.= 0. Hence the required equation is xi t12x3+45x? +64x+30=0. ay oy coy 4,42 ana + satisty () +2) -(2) -{2}:2-0 Hence the apy 6 x) A) WGI NG required equation is 2x*~4x> -3x7+4x+1=0. (4) 02, B2, 7? and 8? satisfy (x)! +a(ui2)? ~ 3x!" ~4x¥242=0. ua, Rearrangement gives x”?(4x—4) = —x? +3x—2. Squaring and simplifying, the required equation is x* 22x) +45x?-28x+4=0. 14 Solution (a) a?:B-y, o.-B?--yand a-B-y? can be rewritten By’, oBy-B and oBy-y. But oBy=3. Hence the required equation has the roots 3ct, 38 and 3y , which satisfy BG (b) a+ B+ =-I. Hence the required equation has the roots ot~1, 1 and y-1 . And the required equation is x7 +3x?—18x-8 which satisfy (x-+1)? +(x+1)? —2(x+1)-3=0. The required equation is x3 44x? 432-350. 15 Solution (ay td and 1 satisfy Qy +2)+ 1=0. Hence the required equation is apy x) 42x07 4150. (b) 0, B* and y? satisty (x! "y +2x"? +1=0. Rearrangement gives x!2(x+2)=—1, Squaring and simplifying, the required equation is 44x? 442-150. ‘SD Solutions Series 18 (c) From (b) 02, B? and y? satisfy x? + 4x? +4x-1=0, and hence =0. And the required equation is 4x? —4x— 16 Solution _ (apt 2 ana + satisty (2) + p{4) +10. Hence the required equation is apy x) PG ro + px?+1=0. 3 2 (b) a2, B? and y? satisfy (x?) + p(x") +r=0. Rearrangement gives = px. ‘Squaring and simplifying, the required equation is x° ~ p?x? ~ 2prx~/? (c) From (b) @?, B? andy? satisfy x? — p?x? -2prx-r? =0. Hence 11 1 = and oP" By satisfy ( i Simplifying, the required equation is P33 42 pre? + pPx- 17 Solution (a) a+B+y=Ya= 3 2 (1b) «2, B?, and y? are roots of the equation (x!) +(x”) +2=0 Rearrangement gives x!/2x = ~x. Squaring and simplifying, x? -x?-4x—4= Hence a? +B?+y?=1. (c) @ 407425 : (since 0.,B, y are roots of the given equation) BP +p?+2=0, ‘SD Solutions Series 9 PrP +2=0. Hence (a +B? +y7)+(a? +B? +¥7)+6=0. From (b) a? +B? +7? =1, therefore 0? +B? +? (4) From (b) @, B?, and y? satisfy x°-x?-4x~4=0. Hence of = (o2), t= (92) and = (72) say (x2) (09) an"? 20. Rearrangement gives x”?(x—4)=x+4. Squaring and simplifying, 22-93? +8x-16=0 act +B* +4 is the sum of roots of this equation. Hence ot +B‘ +? =9 18 Solution (a) a2, B?, and y* satisfy (2!) sax! +r=0. Rearrangement gives x"”"(x-+)=—r . Squaring and simplifying, 42g? +q?x-P =0. a? +B? +? is the sum of roots of this equation. Hence 0? +B? + y? =-29. (b) P+qa+r=0, (since a, B, and y_are the roots of the given equation) BP +gh+r=0, Prqer=o. Hence (a? +B? +y°)+q(a+B+7)+3r=0. Here +B+7 is the sum of the roots of the equation x° +x +r=0= 0+B+y=0. Therefore a? +B? +7? =-3r. (c) @,B,y are also roots of the equation x?(x3+gx+r)=0, i.e. x5 +qx? + rx? Hence a°+go3+ra?=0, B°+gB?+rB* =Oand y+ gy? + ry? =0. Adding these equalities we obtain (aS +B°+°)+q(a? +B? +7°)+r(a? +B? +77) =0. But from (a) 0? +B? +? =—2g and from (b) oF +B +7? Hence oF +B5 +5 = —q(-3r)—r(-2q) = 5qr- 7SD Solutions Series 20 Exercise 4.4 1 Solution P(-i)=i-i+4i~4i=0< (x +i) is a factor of P(x). By inspection, or by polynomial division, x? + ix’ —4x—4i=(x+i)(x?-4). Hence P(x) =(x+ ix-2)(x +2), and these are irreducible factors over C. 2 Solution P(2i) = -8i+ 8i— 61+ 61 = 0=> (x—2i) is a factor of P(x). By inspection, or by polynomial division, x3 —2ix? -3x+ 61 =(x-2i)(x? ~3), Hence P(x) = (x~2i)(x -V3)(x +3), and these factors are irreducible over C. 3 Solution Pix)= ¥(< $lox+ 6+245) = nffe x x eae i 1? 1 Using (-+4) a x24 542, Plxy=x? {x+4) wiq{s+t) Since 0 is not x x x x a zero of P(x), the solutions of P(x)=0 are the solutions of fesfoeda By factorising this quadratic may=2(o+ tet) orob-u?ena?-0r+. ay yeas Hence P(x)=0=> x? +1=0 or 3x7 +10x+3=0 ‘TSD Solutions Series ee a ‘Therefore, the zeros of P(x) are ~: From (1) P(x) = (22 +1)3(x-+3\x-+1/3)= (x2 +(x +3)(3x +1) over R. 4 Solution a(atetssa-2e3)a2 2+ k)e7(x-t}e2 : x x x x . 1V_ ad 2 1y 1 . Using (x-2) =22+4-2, peay=x?{2 x-+) +7 x—1) +6) . since 0 isa x x x x zero of P(x), the solutions of P(x)=0 are the solutions of 1y 1 x2) ofx-2}+5=0, x x By factorising this quadratic, ; roo eaf{ -2}eah{( ~t) Sher earner sar-9). o 7 ee Hence P(x)= Plx)=0= x?+2x-1=0 or 2x7 +3x-2=0, -14V2 ans ~Dors=4. 2 Therefore, the roots of P(x)=0 are 24-188, From (1) P(x)=(x+1-V2)(x+14V2)(x+2\2x-1) over R. 5 Solution 6 =e DGS txt e+e) = (e-DfxGat ta? Dot +? 4D} =(x-Drt Dt +27 +0). Hence x°~1=0=9 x =41 orx* +x? +1=0. Further more, the sixth roots of unity are equally spaced by a around a circle of radius 1 and centre (0,0) in the Argand ‘78D Solutions Series 22 diagram. The zeros of P(x) = x* +x? +1 are the non-real sixth roots of unity which are: = BS cin zyand = %, where z= cos +isin ea Oneeeaon zs ands =%, where z= cos E+ isin, Hence ‘These factors are irreducible over R. 6 Solution Let Q(z) = x° +1, then Q(+i)=0 and hence (x i)(x +i) = x? +1 is a factor of Q(x). By inspection, or by polynomial division, x° +1=(x? +1)(x4 — x? +1). Hence x+150=> x=tiorx*—x?+1=0. Therefore the zeros of P(x) = x* x? +1 are the solutions of x= I x#ti ‘The sixth roots of -1 are equally spaced by z around a (0,0) in the Argand diagram, The sixth roots of =I different from +i are: x © icin ® 25% =, Where 2, =cos™ +isin™, = 5m gin SE 24,25 = %y, where z= cos—Z + isin =H Hence SD Solutions Series P(x)ax4 =x? 41 = (x- aa Z)(x- e424) 23 (x2 -2Rezpx +|zpf)(2x?-2Rezyx tle) = (22-2008 E4iyex?-2008Ex4) 6 Using cos: ‘These factors are irreducible over R. 7 Solution xto tia (v3 r4(x? V3 x4). 25-42 =0= 2(z4~4)=0. Hence z=0 or z is 2 complex fourth root of 4 . Clearly two such roots are 4/4 = +2 . The other fourth roots of 4 are equally spaced by $ around a circle of radius 2 and centre (0,0) in the Argand diagram. The fourth roots of 4 are 29 = V2, 2 = V2, 4 = V2i, 2 Hence z°—4z=0 has roots 0;-V2,+¥/2é. -V2i. 8 Solution 4S+z2=0=> e(<*+4)=0. nee z=Oorz is complex fourth root of -t. Clearly one such & 1 foot has ar; it = and moduh + Since 1 gument 7 and modulus “7. s < we(-4)- ac 4 -(4) The other fourth aN | roots of -t are equally spaced by a around a circle of radius z and centre (0,0) in the Argand diagram. 1 The fourth roots of + are Fond nt) and leo ising ‘SD Solutions Series ), 24 Hence 42° +z =0 has roots 0, ata, F-12) 9 Solution Let z=cos6+isin@. Then by De Moivre's theorem, z’ = cos 40+ isin4@. But by 4 the Binomial theorem, z* =(cos@ + isin 0)* = ¥ (i) if sin* Ocos*~* @. Equating real io) parts, 0546 = cos* 6 - 6cos? Osin” @ + sin’ 6= 8.cos* 8—8cos"8+1 (a) Let cos = x. Then cos40 = 0 > 8x4 8x? +1=0. Hence if @ is a solution of cos40, cos@ is a root of 8x*—8x? +1=0 But cos4f O=>40 25+ 2m, n integral 8 + + Enna 0ehe2.K These values of @ give exactly four distinct values of cos, namely ieee 9 m3 3 cos ,cos=m = —cos— 2, cos—% = —cos—~,cos—m = cos—T. cues 8 8 8 At the same time considering 8x* ~8x? +1=0 as a quadratic in x? 42v8 _ 242 ‘SD Solutions Series 28 These values of 6 give exactly four distinct values of cos@, namely cos ,cos *n,008-Ln =~cos n,c0s3n = —cos® R12 12 a1 12° At the same time considering 16x‘ —16x?+1=0 as a quadratic in x”, 82va8_2+V3 re 6 4 rath ioe orx=t3 2-3. TS oog5t But x=cos0. Since cos => cos7>0, we deduce that FD cos B= 1 2- V8. 122 10 Solution Let z=cos@+isin®. Then by De Moivre’s theorem, z*=cos5@+isin50. But by 3,(5 . the Binomial theorem, z= J) () i sin* Ocos>* @. Equating real parts, bol c0s50 = cos* @—10sin? 6cos? @ + 5sin* cos ® = 16cos* 8 20cos"0+5cos0. (a) Let cos@ = x. Then cos5@=1 ¢> 16x°—20x° +5x =. Hence if @ is a solution of cosS8=1, cos@ is a root of 16x°— 2033 + 5x— 2 =0. But cos50 =1=> 50=0+2mm=6 ma, n= 0,t142,K 5 Since the period of the function cos¢ is 2m, this formula gives under n = 0,1,2,3,4 all the values of cos, namely cos ,cosSn= cos? 5S 2.4 RE O.6 c0s0 = I,cos=1,cos m= —cos-~ cos m 5 5 5 But cos®= x and hence 16x° - 20x? + 5x — =0 has roots 1,c052n,cos=n,cos+n,cos tn See Se | Let cos2n=a, then cost 5 5 16x5 - 20:3 + 5x But 4 8 and cos 22 —1+98) _ eee Saale 9 16x5 2023 +5x=4. Hence if 6 isa 2 (b) Let cos@ =x. Then cos50 = solution of cos5@=—~, cos@ is a root of 32x5—40x3+10x~1=0. But cos50 = 3° 50=22+2nn, nintegral 0= n= 0,t1,42,K These values of @ give exactly five distinct values of cos@ , namely cos cost m,cost2 cost? cos on = cost at. + Siete fea eee 32° (i) The sum of roots 32x°—40x3 +10x—- is zero, hence ; peal x (ii) The product of roots is 35, hence cos cos 1319 RCOs— Neos —T Orie m7 1319. cos FCos T= teos, = Reoss= Stags S tees 15 eam) ‘SD Solutions Series ” Exercise 4.5 1 Solution , 2x+10 @-DGE+3) —2_ Then 2x+10=¢,(x +3)+cy(x—1). Putting x x+3 gives c =3, while x=—3 gives c) = 2x+10 Hence —~**0 ___3___1 “ G=DE+) x 2 Solution Using the quadratic formula, we get 2x? +5x+ a=ax #3 )e+)= (2x+3Xx+1). 4xt+5 a, & ES tg 2 then 4x4 5=ay(x +1) + 09(2x +3), Patt Gere Deed xt eM AE HSH a+ D+ eale+5), Putting x=-1 gives ¢ =1, while x =-3/2 gives ¢ =2. 4x45 2) 1 Hence —~——— = . 2x? +Sx+3 2x43 x41 3 Solution Using the quadratic formula, we get. 2x? —Sx-+2=2(x—1/ 2(x—-2)=(2x—1)(x-2). Let ae — 6=¢(x-2)+cp(2x-1). Putting x=2 gives cy =2, while x=1/2 gives q =-4. 6 en 2 He = TT 2 om Fee Sx42 Dx-1 x-2 4 Solution 2 2 By divisions cee ee ge eee xG4D tte 4x +l) Let —2 2 Then 2=4(x+1)+cyx . Putting x=0 gives c =2, while x(xtI)ox x41 x=-l gives ¢ =-2. ‘SD Solutions Series 5 Solution 2x44 q_atb + een (x= 207 +4) x72 Ped - Then 2x+4= o(x? +4) +(ax+b)(x~2). Putting x=2 gives ¢ Equate coefficients of x7: O=q+a=>a Put x=0: then 4=4¢,-2b=>b=0. 2x44 1 x Hence 3 = (20744) (2) GPA) & Sofution re 3x?-3x42 cared Qx-DQ? + 2x Ther xe 3x? -3x4+2= cx? +1)+(ax +b)(2x—1), Putting x=1/2 gives c Equate coefficients of x?: 3=q+2a=>a=1 Put x=0: then 2 b> b= Bx2-3xt2 1 xn x+1 Hence —*— = (2x=x? +i) 2x- 7 Solution It is necessary to perform the division transformation before seeking partial fractions. By division, X42 46x41 +2 Gx HIO_ |, HIE yx H246 GNC +4 Pa earth Pte earth | (eta? +4) x242x4+6 Garth (eA? +4) tt x4 Let Then x7 421 +6=.0(x? +4)+(ax +b)(x4+1). Putting x=—1 gives c, Equate coefficients of x7: 1=q +a=a=0. ‘7SD Solutions Series 29 Put x= then 6=4¢ +b=2 b=2. 3 2x? +6x+10 1 =l+ | aa ara G+DGt+4) + 8 Solution Let —2—* SBD then 5-2 = 6((2?-+1)+ (ax +b) 2x43). — + (2x+3)(x7 41) 2x43 x? 41 Putting x=-3/2 gives ¢ =2. Equate coefficients of x7: 0=c,+2a=>a= Put x=0: then 5=¢,+3b=95=1. ~ —2=%__ 2 (2x43)(x? +1) 2x43 xP 41 Hene 9 Solution +7 axtb oxtd GP +I +4) P41 x? 44 x47 = (ax+b)x? +4)4 (crt d(x? +1). Equate coefficients of x? Equate coefficients of x : Equate coefficients of x? : Equate constant terms x47 21 Hence —* + 7_ : (GP 4D? +4) 241 x44 40 Solution 3x axtb oxtd : ; a -Then 3x =(ax+b)(x? +4) +(cr+ d(x? +1). Gaede eg NMEA FO Hort ae HD) Equate coefficients of x?: O=ate Equate coefficients ofx : 3=4a+c Equate coefficients ofx?: O=b+d Equate constant terms =4b+d, Joe ).d =0. ‘TSD Solutions Series 3x Hence Cited) | ‘78D Solutions Series 31 Diagnostic test 4 1 Solution (i) In order to solve the polynomial equation P(x) = x*—4x? +3=0, denote x? ast and use the quadratic formula. Then x? =1 or x? =3. Hence P(x) = (x? -1)(x?-3). (a) Inreducible factors of P(x) over Q are P(x) = (x~1)(x+1)(x? -3). Each linear factor gives rise to a zero of P(x). Hence the zeros of P(x) over Q are +1. (be) Irreducible factors of P(x) over R and over C are P(x) = (x= I(x +1)(x— V3)(x + V3). Hence the zeros of P(x) over R and over C are 41,43. (ii) Use the quadratic formula, then P(x) = x4 —2x?-3=0=5 x? =-1 or x? Hence P(x)= (x? +1)(x7-3). (a) P(x) has no linear factors over Q and P(x) =0 has no solutions in the field of rational numbers. Hence P(x) has no zeros over Q. (_b) Inreducible factors of P(x) over R are P(x) =(x? + 1)(x—¥3)(x+3). Bach linear factor gives rise to a zero of P(x). Hence the zeros of P(x)over R are +V3. (c) Inreducible factors of P(x) over C are P(x) = (x—i)(x +i)(x-V3)(x +3). Hence the zeros of P(x) over C are +i,+ V3 2 Solution (a ) If @ is arational zero of P(x), then ct is a divisor of the constant term. Hence the only rational zeros of P(x) are +1,45,+10. By inspection, P(1)=0 and P(-5)=0. Hence (x—1) and (x+5) are the factors of P(x). Dividing P(x) by (x-Ix+5)=x?+4x—5 we obtain P(x) =(x—1)(x+5)(x? +2), and these are irreducible factors over Q and R. (c) Irreducible factors of P(x) over C are P(x) =(x-1)(x +5)(x—¥2 ix + ¥2 i) ‘7SD Solutions Series 32 3 Solution P(x) = 4x3 +1527 +12x—4, Pox) = 12x? +30x +12, =9 P(-2) 0, P(-2)=0. Hence -2 is a double zero of P(x) and P(x) = 4(x+2)"(x-+2) for some constant k, as P(x) is a polynomial of degree 3 with the leading coefficient 4. Then P(0)=—4=9 k==1/4 and P(x) = (x+2)*(4x—1). The zeros of P(x) are -2,-2,1/4. 4 Solution P(x) =2x3—x?—6x+3. All rational zeros of P(x) have the form p/q , where pandg are integer divisors of 3 and 2 respectively. Hence the only possible rational zeros of, PUX) are +1,+3, +3/2. Butof these, only 1/2 satisfies P(x) =0. Hence (2x-1) isa factor of P(x). By polynomial division, P(x) = (2x-1)(x—3) = (2x -1)(x--V3)(x +-V3),, and these are irreducible factors of P(x) over the real numbers. Each linear factor gives rise to a zero of P(x). Hence the zerosof P(x) are 1/2, +3. 5 Solution (a) xi isa linear factor. Hence we can use the remainder theorem, and the remainder is Pli)= =o.) 3-1 (b) P(x) = x3 +2x?-1 and D(x) =x? +1 are polynomials over Q. By the division transformation, P(x) =(x? +1)Q(x)+ R(x), where Q(x) and R(x) are polynomials over Q, such that deg R< deg D = 2. Thus P(x)=(x?+1)Q(x)+ar+b, a,b rational, and this equation is true for all x €C. Then Pli)=-i-2- -3-i=-3- ai+b. But aand b are real = a=-1,b= Hence the remainder is ax +b =-x—3. ‘SD Solutions Series 33 6 Solution P(x) =x‘ + ax? +bx . By the division transformation, P(x) = (x? +1)Q(x)+x+2. Then Pli)=1-a+ bis l-atbizit2 -a=2,b=1 2a}! a=2, 7 Solution P(x) has real coefficients. Hence P(1- i) =0=> P(I+i)=0 and then [x-C-a][x-+)]= x? -2x+2 is a factor of P(x). By polynomial division, P(x)=(x2-2x+2\(x?—3). Hence P(x) = (x? ~2x+2)(x—V3)\(x +3), this is the factorisation of P(x) into irreducible factors over R, and P(x) has zeros 1+i, +3. 8 Solution P(x)= x‘ +ax? +6, as P(x) is an even monic polynomial of degree 4. Then P(\2)=0=9 4+2a+6=0. Hence a=-5 and P(x) =x4—5x7+6. P(x) is even. Hence P(J2)=0 => P(-V2)=0 and then (x - 2)(x +2) = isa factor of P(x). By inspection, P(x)=(x? ~2)(x? ~3). So the irreducible factors of P(x) are P(x)=(x-V2)(x+V2x-V3\(x+3). 9 Solution Let the roots of P(x) =x? 3x? +ax+8 be c—b,c,c+b. Then sum of roots is equal to 3, hence 3c = 3=) c=1. Product of roots is -8 => 1—b? (that gives the same roots of P(x)), Hence the roots of P(x) are -2,1,4. Therefore LaB= =-8=b=30rb=-3 ~8+4=~6, hence a=—6. ‘SD Solutions Series 34 10 Solution 3 ye x) (x x =) (2) -of2]-3= (a) 204, 2B and.2y satisfy (3) +3) (2) 3 is Hence the required equation x8 +2x? -8x-24 = 0. (b) 28 and Z satisfy (2x)'+ (2x)? ~2(2x)—3=0. Hence the required equation i 8x3 44x? -4x-3=0. (©) a—2,B~2 andy ~2 satisfy (x+2)° + (x+2) -2(x +2)-3=0. Hence the required equation is x? +7x?+14x+5=0 (4) @+2,B+2 and +2 satisfy (x-2)° +(x-2)* -2(x-2)-3=0. Hence the required equation is x? —5x?+6x-3=0. 11 Solution 3 (a) it and | satisty (2) +q2}+=0 Hence the required equation is Y x rbtgx?+1=0 3 (b) 02,8? and 7? satisfy (x”) +gx"? +r =0. Rearrangement gives x!(x4q)=-—r. Squaring and simplifying, the requited equation is 24 2gx? +q?x— 12 Solution (a) a+B+y=0, as the coefficient of x? is zero. 3 (’b) o2,B? and y? satisfy (x”) +2x" +1=0. Rearrangement gives x"2(x42)=-I. Squaring and simplifying, x° + 4x? +4x— «Hence the sum of the roots of this equation a7 +B +y72=-4 ‘78D Solutions Series 35 (c) a +2c0+1=0 (since «,B,y are roots of the given equation), BP +2B+1=0, pP+2y41=0, hence (a? +8 +y')+2(a+B+y)+3= - But from(a) 0+B+y=0, and +p +y (d) From(b) a, B? and y? satisfy x7 +4x?+4x-1=0. Hence a*, B* and y* satisfy (ery +4(v2)° 4x? =0. Rearrangement gives x/?(x+4)=1-4x. ‘Squaring and simplifying, x? -8x? + 24x « Hence the sum of the roots of this equation at spay = 13 Solution P(x) has symmetric coefficients, hence it can be converted to quadratic equation in =) P(x) = 3x4 — 4x3 — 14x? ~tee3ea'fa? +5 -a-d ad x x {Hees} Aer3)->f sme 0 is not azero of P(x), the solutions of P(x) are the solutions of 2 art) -o{x+4)-20- x x Poy=s9{x+t)-s0l(2+4+2) = (00? 1053)62-425-1). x x Hence 2 Using («+4) =x2424+, we get (x)= x x . By factorising this quadratic P(x)=0=> 3x? -10x+3=0 or x?+2x+1=0, and stVi6 3 Ir ‘TSD Solutions Series 36 So the roots of P(x)=0 are -1,-1,1/3,3, and from P(x) = (3x? =10x+ 32 +2x- 1) it follows that the factorisation of P(x) over R is Plx) = Bx=I(x-3a +1? 14 Solution 162 =0=9 2(z* 16)=0. Hence z=0orz is a complex root of 16. Clearly, one such root is 2, as 24 = 16. The other three roots are equally spaced by a around a circle of radius 2 and lee center (0,0) in the Argand diagram. The fourth \4 zs roots of 16 are +2 and +2i. Hence z* the roots 0, +2, +2 16z has 15 Solution Let 2 =cos@+isin®. Then by De Moivre's theorem, z° = cos30+ isin30.. But by the Binomial theorem, :05° 0 + 3icos? @sin @ -3cosOsin” @— isin? @. Equating real and imaginary parts, cos30 = cos® @ — 3cos@sin*@ and sin3® = 3cos"0sin @—sin76. 3cos*@sin@—sin?@ _ 3tanQ- tan? cos?@-3cos@sin?@ — 1-3tan?6 Dividing one by another, tan30 Furthermore, it is clearly that the equation P(x) = 3x+1=0 has the integer root ~1. By polynomial division, x? ~3x? =3x +1=(x+1)(x?~4x +1). Using the quadratic formuta, the roots of the equation are -1, 24/3. 1 0=, then 30=~ and tan30=1. 12 4 ona Hence tan3@ = 38 0— "80 9 _, tan? 9 3tan?@—3tanO-+1=0. Lee x= tan, then 1-3tan*@ x tan? Q—3tan?@—3tanO+1=0 9 x°-3x7~3x+1=0. But ock ‘ISD Solutions Series 37 Then 0< tan i= tan 22-3 Su _m Sm 1 1 243 Now 5827 oy pgn S21 __ 2 la, Ow a 212 12 tann/12 2—W3 (2—-v3X24V3) 16 Sotution Let z=cos@+isin@. Then by De Moivre’s theorem, z° = cos50+isin50. But by the 5.(5 Binomial theorem, 2°= 5 ( i sin* @cos** @. Equating real parts, a 0550 = cos’ @—10sin? @cos*@ + Ssin*8cos@ = 16cos°8 - 20cos? 6 +5cos0. Furthermore, 16x°-20x° + 5x =0 = x(16x‘ — 20x" +5)= 0. Hence x=0 or 16x*-20x7+5=0. To solve the last equation use the quadratic formula in x”, then x? = stv5 So the roots are 0, SHB : — Let x=cos0. Then 0550 = 049 16x°- 20x +5x=0. 16 But 0<.<2® cos > 0=> cos 10 10 17 Solution Using the quadratic formula, x? — x -6 =(x-3)(x +2). 2x Ae ; Let Fayed) tag: Then 3x4 = G(x+2)+ q(2~3), Putting x gives 3x-4 1 2 Wox-6 x-3 442" ¢=2, while x=3 gives ¢ =1. Hence ‘TSD Solutions Series 38 18 Solution , Be 6r +10 c_,axtb (e=HGP + x4 P41 Then 3x? -6x+10= (x? +1)+(ax+b)(x-4) Put x=4: then ¢=2. Equate coefficiems of x*: 3=c+a=a=l. Equate constant terms: 10=c-4b=>b 2 3x? = 6x +10 Hence ———>— (2-4)G° +1) ‘SD Solutions Series 39 Further questions 4 1 Solution Let P(x)=2x3 ~13x—7, then P(W7) =14V7 -13V7 ~ V7 =0. Hence x- V7 is a factor of P(x). By polynomial division, P(x) =(x-V7)(2x? +2V7x+1). Factorising 2x7 +2V7x+1, rayaees{x fi 2x+4/5 +7). Hence the roots of the equation P(x)=0 ae 7, EE 2 Solution = (x? 1005+. x4 +27 41), Hence @y Itis clear that x* x8-1=0¢927 6 +x4+x?+1=0. So the zeros of P(x)are the 0 or P(x) =. solutions of x* =1, x#+1. Clearly 28 =1=92 is acomplex eighth root of unity. These roots are equally spaced around the unit circle in the Argand diagram, the angular spacing being 2 real eighth roots of unity ase zy and %, where z; = eos thin, zz and 23, where z) =i, = Ri 23 and z5, Where z3 =—cos + isin Hence P(x)=0 has the roots pit. ti, pcs) and P(x) = (x= 4a AMx- AM -HVx-Bx-H) = (x? ~2Re gx +[e\[" (x? -2Rezpx + [eof Mx? —2Re zyx +]e4?). ‘SD Solutions Series 40 Hence P(x) = (x? -x¥2+1)(x2 +(x? +2V2 +1) isa full factorisation of P(x) over R. 3 Solution ‘The quatic equation P(x)=5x* —1 1x? +16x? ~1 Lx +5 =0 has symmetric coefficients 1 and so it can be converted to quadratic equation in (« +=}. x Pix)= 2{5¢x7 $132) 11 41/2) 416} = xf 5x41 x)? = 1G 4 1/2) +6} Since 0 is a not a zero of P(x), the solutions of P(x) are the solutions of 5(x+1/ x)? 1 I(x +1/x)+6=0. By factorising this quadratic, roa s{retoil{se(x+t) 6} G2 —140e?—6r+5) Hens x x P(x)=0e9 x7-x+1=0 or 5x?~6x+5=0, and 2¥Bi gy 2 5 x So the roots of P(x) =0 are faa i, 3G# 4i). Since these zeros are non-real, the full factorisation of P(x) over R is P(x) =(x?-x+1)(Sx? -6x+5). 4 Solution Let z=a+ib,b #0 and P(a+ib)=0=9 P(z) = P(a—ib)=0. a+ ib isa double zero, then a~ ib is also double zero too. Since P(x) is a monic polynomial of degree four, - = 2 Pla) = (2~ 2x2) = f(x 2x DY = (x? -2Reze +f) = x?—2ax+(a? +5?) = x4 —4ax? + (6a? +B )x? + daa? +b?)x +(a? +b), But P(x) = x4 —8x3 +302 56x44 Equate coefficients of x*: -4a=-8=9a=2. Equate constant terms (a? +b)? = 49 => 5? Hence the roots of P(x) are 2+-/3 i, and the irreducible factors of P(x) over Rare ‘TSD_Solations Series 41 P= Prax + (a? +03} =(2? 447, 5 Solution Let P(z)= z*+327-62+10, then P(L+i)=(1+4i+6i? +47 +i4)+3(1 + 21+ 7)-6(1 +i) +10=-4+ 61-6 -61+10=0 P(z) has real coefficients, hence P(1+i)=0=> P(I+i)= P(I-i)=0. Then {2—(1+i)}{z—(T+i)} = (2? - 22 +2) is a factor of P(z). By polynomial division, P(2)= (2? 22422? +22 +5). Using the quadratic formula, 27+22+5=0 => 142i. Hence the roots of P(z)=0 are 1+i,-142i. 6 Solution xtvx?-4 Let x= yt yPtsy41=0= yt or y 1 1 1 : Then a+—,B+— and y+— satisfy awe p md yy satsty x#(2? 4?) wt(x?—-4)" +3 425 2 2 a 2 27,2 3 a . te Rearrangement gives #(x?-4) "(4x2 +8) =—4x° —16. Squaring and simplifying, we get 2x3 +3x7+8=0. 7 Solution The sum of roots of the equation x*— px? + qx? ~ pgx +1=0 is equal to atP+y+8=p. Then ‘TSD Solutions Series a2 = (04B+YatP+dMa+y+8)\B+y +8) =(p—BX—WP- BPA). Expanding, t= p*— p(a+B+7+8)+ p?(oB+ oy +08 + By +B +18) — ploy + 085 + oy5 + By6) + aByS But Ya=p, Lop=49, LoBy= pq, aBy5=1. Hence 2 t= p*=p*+ p'g- p'gt 8 Solution Let P(x)=x4~ px3+gx?—rx4s. Then P(x) = (x= ax BN WWx~8)= [x= (+B) + OB} C+ B)2 +28} (a+ Bry +B)x + {(a+B)(y +8) + OB + 98}? — {aBly + 8) + 76(a.+B)}x + oBy6. (a) Equate constant terms: oy = s. But oB = 76 => (a8)? =. At the same time the coefficient of x oB(y +8) +78(4+B)=Of(A+B+y+8)=OP-p, as a+B+y+5=p. Equate coefficients of x: aB- p=r=> 1? = (of)? p? = r* = sp? (bd) Equate coefficients of x*: a+P+y+5=p. But at+p=y+5>a+B=p/2. Equate coefficients of x2: (a+B)(y+8)+0B+y6=q= oP +y5=4-(p/2). Equate coefficients of x: of(y+8)+76(0+B)=7 = (af + y8)(a+B)=r—> 7 {r-(8) |oars p>—4pq+8r=0. 9 Solution Let O(z)=2"" +2" 24K +2 +1. Then Q()=n. Furthermore, 2” = (2-12! +2" 24K +z +1). Hence the roots of Q(z) = 0 are 2,22,K Zn) Therefore Q(z) =(z~ 2 z~z2)K (22,4) and Q(1)=(1— 41 22)K (I= 2,5) But QM=an = (1-yX1-2)K (I- gy) =n ‘TSD Solutions Series 4B 10 Solution Let P(x) =x? +3px+3qx +r. A double root of P(x)= must be a single root of P(x)=0. Then Pi(x) = 3x? + 6px+3q=0=9x=-pt wLet e=4l, k= , then x=-p+ek. Let us calculate P(-p+ek): x3 =(-pt+eky =—4p?+3pq+(4p"—q)ek, 3px? = 3p(—p+ek)= 6p? —3pq-6p"ek, 3qx = 3q(—p+ek)=~3pq+3gek. Hence P(—p+ek)=2p?—3pq+ek(—2p? +2q)+r. But it must be P(—p+ek)=0. Therefore, €k(-2p* +2g) =-2p?+3pq—r. Squaring, (p?—9)(4p* + 4q? 8 p2q) = 4p® + 9p2q? +r? - 124g + 4p'r - 6 par. This is equivalent to Ap? — a)? +(p? — q\(4p* ~8p"q) = 4p° +9 pq? + 7? -12p"q—2pqr +(p?—g)4pr Rearrangement gives 4(p? - gq? = pr)=-(p?- q)(4p4 -8p"q) +4 p° +9 p°q? +r? — 12p*q—2par . ‘Simplifying the right hand side of this identity, 4p? —q)\q? - pr) = pq? -2par+r. Hence 4(p?—q)(q? - pr)=(pq-r)?. 11 Solution Let P(x)=x" + px—q.Then P(x)=nx""! + p. Hence P(x) => x"! =-p/n. (Remember, that ot is a double root of P(x)=0 if and only if P(a)=0 and P(a)=0, but P’(cr)#0). Furthermore, P(x) =0 9 x(x"! + p)=q. Substituting x" =—p/n, we obtain Pa pleg = x=—"4—. But x" =-p/n n (a-Np 7SD Solutions Series 44 Hence { ad (n=Dp, 12 Solution Let P(x)= Ya,x"" =x" +ayx"" + ann” 74K +0, 4x +0, mm Then P(x)=(x-1)(x—2)K (x—n), as the roots of P(x) are the first m positive integers. Hence a, =-La=-Fk=-F (n+) and a, = [a= [=n 7 il At the same time %=DoB= Dkl. Iskelsn In order to calculate this last sum, we use the following identity 2 (E] ap+2 Yaya, where a, eC, k=1.Kn. il fei iskelsn 1 But k= int nine), $2 = ROEDER D Hence a, ist 6 So a, = Pmt Ena DQne = n(n + 1{3n(n+1)-2(2n-+ D} 24 (n +1)(3n? —n- 2) =hnn ¥1)(n-1)Gn+2). ‘SD, Solutions Series ERGO Laskar seer F 2 7SD Solutions Series Worked Solutions to Popular Mathematics Texts Suggested Worked Solutions to “4 Unit Mathematics’’ ( Text book for the NSW HSC by D. Arnold and G. Arnold ) Chapter 5 Integration 411096 %, J , ‘ on co 2 mau ew ae AM iL Solutions prepared by: Michael M. Yastreboff and Dr Victor V. Zalipaev 7SD Solutions Series Worked Solutions to Popular Mathematics Texts INDEX page Exercise 5.1 1 Exercise 5.2 5 Exercise 5.3 12 Exercise 5.4 17 Exercise 5.5 20 Exercise 5.6 26 Diagnostic test 5 28 Further Questions 5 33 Solutions are to “4 Unit Mathematics” [ by D. Arnold and G. Arnold (1993), ISBN 0 340 54335 3 ] Created and Distributed by: 7SD (Information Services) ABN: 13009821 © 780 1997 ‘The 7SD team welcomes all feedback. Correspondence should be addressed to: 7SD attn: Michael Yastreboff POBox 123 Kensington NSW 2033 Exercise 5.1 1 Solution , Using the pattern [fOu =ln|f(xo|+e with f(x) =1+2", we have x 2xdx teen tines" Sata ghar gilt tes gin te, since 1+x?>0. 2 Solution Using the pattem f(F(OF'/"nde=—L (fy +e, n=—2 with f(a) =142%, wehave x 1, 2x 1 Ste ay ae 3 Solution The given integral follows the pattern fe!“ f’(x)dx =e! +c with f(x)=sinx, and we have Je" cosxdx = eM +0. 4 Solution ‘The given integral follows the pattern [sin{ f(x))f“(x)dx =—cos(f(x)} +e with f(x) = and we get Jet sin(e*)dx = —cos(e*) +c. 5 Solution ‘The given integral follows the pattern Jers @rde= yey +e with 7 f(x)=1+27, n=1/2, and we have fx fie Pac= 3 f+)? ands = Lasse, 6 Solution a 1 een XZ) a The given integral follows the pattern f- Terre sin’ re with a=2, and we get [Se ares Spree Gre. ‘78D Solutions Series 2 7 Solution Ltd 1 Gian +e with a=, and we get The given integral follows the pattern Jean fran taxte. 1 4 8 Solution r 3 1 The given integral follows the pattern J(f(x)} f"(x)de=—{f(2)}" +e with f(x)=tanx, and we get Jian? xsec'xde = $ttan x} +e. 9 Solution Using fsec’(f(x))f(x)dr = tan f(x)) +6 with f (x)= 2°, we get Jxsec"(x?)dx = sec*(x?)2xdx = prance? te. 10 Solution ‘ Using Je! f"(x)de =e! +e with f(x) = Vx, we have Jae eR lydr=2e7 +e. I 11 Solution Using the pattern f(f(2)}" f/(n)dx = aye +e with f(x)=cosx and n=—4, we get xdx (cosxy? sec?x Jsectx tan xdx =f cosxy4(-sin x)de = = +e. cos'x 3 3 12 Solution Using JF Pa mtalgooj+e with f(x) =sin? x+2, we get _sindx py, _ f2sin xe0sx 4 =Inin? x +2]-+c=In(sin® x+2)+c, since sin?x+2>0, laa a Sasints x SD Solutions Seri 13 Solution Using the pattem f (f(x) f"Ga)dr= Inf] +e with f(x) =sin x, we get In(sin-Z)—In(sin =) 14 Solution Using the pattern fcos{f(2)}f "(dx =sin( f(x)} +e with f(x) =Inx, we get feosana)4ae =sin(In x)f =sin(In e)—sin(In1) = sin(1)—sin(0) =sin1. } x 15 Solution Using the pattern f a aex 2 16 Solution Using the pattern Sage Yai-a)| a tt = In| Lea 17 Solution +e with a=1, we obtain e Pf = In(3+-VB)~In(y/3-+-Vi = In 4242 = inc V2) a V2+1 1 Using the pattern \ee* =In{x+¥x? +a?) +0 with a=2, we obtain +a ad =[In¢x+ Vx? +4)] = (2+ V8) In(2) = Incl + V2). z ft la 7SD Solutions Series 4 18 Solution Using the patter [——ade=sin "= with a=1/2, we obtain (a? —x*) G Loa dint =Z zie =F 19 Solution Using the pattem J (f(O) /’@)de=AFG)F' +e with f(a) = cos2x, we get tan 2xsec2xdx = [— ote 20 Solution Using JU@r side = In|f(x]+e with f(x) =1+e%, we have leer flog +e? = ind-+e")-In(l +e) =In4—In2=In2, since 1+e*>0. ‘78D Solutions Ser Exercise 5.2 1 Solution It is clear that x? +2x+2=(x+1)*+1. Make the substitution x+1=u, dx=du. Then we get 1 i . IF Gara [ duet “1 stan utestan x41. x 42x42 Sia 2 Solution Skis clear that 2x—x? =1-(x~1)?. Make the substitution x~ , dx = du . Then we get 1 1 du rl ae So = sin ute=sin\(x-I)tc. aoe oF "ioe 3 Solution It is easily seen that x de 2x a le leee. ti de =[——ae-| Ede fed ings? +1) - tan“! +e, since att zat le G? +1 22x. 4 Solution It is easily seen that x(2x+1)_(x+1-I2x+2-1) _ 2x41) -3{x+1)41 x+l xtl oe L 1 =2c41)-34 22-14, Ce riemeees ‘Then we have fC) arn fer te yarn faxde— fader [oat 2_x+mnfesl|+e. 5 Solution Let x+l b (2x4i)x x 2x41 . ab constants. Then x+1= a(2x+1)+bx. Put x=0: a=1. 7SD Solutions Series axel dxf ae = tn} - Linfax +1]+¢= Inf Ll | x4) Soa Ha-ginke= + \Wax+il)** 6 Solution : x 3x+2 By division TT ara) | G@FD@+2) bre? a,b (eDG@+2) x41 x42 Then we get 3x+2=a(x+2)+H(x+1). Hence j = fde—{—2#2 aera GrG+2) = dx —4| aia Waa* 7 Solution Iris easily seen that Qxt3=2x4+2+1= (x? 42x45) 41 Hence 2x43 (2242x457 1 2 a dx =In|x? +2x+5|+,———. Varroa “Fa 22x45, eaeoees f a levee = Ina? +22-45)+2 tan bee, since x7+2x+5>0, xe R. 8 Solution , —Sx=10 a (2+1e+3) xt x3 ‘SD Sotutions Seri ‘Then ,we have 6x~10m a(x~3)+b(x+1). Put x=-l:a=4. Put x=3:b=2. Sra10 yf 44 2) see Seog sites =4ln|z +1|+2Infx—3+e=In((x+(x-3)") +0. 9 Solution Make the substitution x—1=u, dr = du. 4 1 1 Then [———~dr = 4f = ———_ar = 4, —1,—. wn faa leaned ear ar hb apg A pa pte) Fifer pe Poe He, jut 2 x—1+¥2| Then 4x—x? = a(x? +4)+(bx+c)(x+1). Put x= =Sasa= Equate coefficients of x?:-1=a+b=9b=0. Equate coefficients of x!: 4=b+ce=9¢=4. Hence xn. 4 -1 4 A tae Jae = J de [A ae = -infe f+ 2tan Wye. G+DG? +4) CS vat Set aga et tea Ge 11 Solution 10 a bxte Let —°__. 7 (e102? +9) x1 x7 49 ‘Then 10= a(x? +9)+(bx+eMx-1). ‘SD Solutions Series Equate coefficients of x7: O=a+b. Equate coefficients of x': O=c-b Equate constant terms: 10=9a-c. Thus we get a=1, b=-1, c=-1 Hence i 10 aj (x=)? +9) = nfs 1-39 +9) since x7+9>0, xER. 12 Solution 3 axth oxtd Let =—— : CP +NG7 +4) x41 x44 Then 3=(ax-+b\x?+4)+(cr+d\x? +1). Equate coefficients of x°: O=a+c. Equate coefficients of x7: O=b+d. Equate coefficients of x': O=4at+c Equate constant terms: 3=4b+d. ‘Thus we obtain a= Hence 3 1 1 ap 1 = ir =f def) ae = tan! x1 ae } I ae iaea sera eeameie Dial 13 Solution Using the substitution x — ‘SD Solution 14 Solution Using the substitution x-+1=u,de = du, x =0=9 u=1,x=-1=9 u=0, we get 9 1 _¢ 1 a 1 a Vina 1 Ja— cri 1M 1 = i =sin"M1/2)—sin@)= 2, 15 Solution 2 2 2 2 tee] Goes sai gee= Hina? eof Afni] ots gx th gx td 2 2 2. 1 (in8~In4) +2 ¢an"\0)~ tan“!(0)) 16 Solution 95? By division 242-2" -y434—1 2=x 2=x Laay-ox? 1 Then f X=" ge =f (2x43 yde+ f de 3” =x a a2-* 2? +30] -[ine- aff, =4-(In1-In2) =44In2. 17 Solution Make the substitution x— =u, dx= du, x=1=9u=0,x=2u=Lx=utl Hence 4 2x-3 4 2x3 1 Qu-1, 7 _2u 4 1 du J te = § Ste = f Stu = | Se -J ° Ina? +0), - tan! f px-2xt2 0 (e-IP HT og wl og 4 wel In2~In1- (tan! 1 tan“ 0) = In2- 18 Solution 2 By division 24245. 2 Gana GrDa+s) 2 a Let = — (x4 DG+3) HT) (+3) a,b constants. ‘SD Solutions Series Then 2=a(x+3)+ (x41) Put x=-k 2=2a>a=1. Put x=-3 2=-2b=b ‘Thus we get a 2eaees 2b 2 a1 ba f AS ae f ude | are bh | oat) Sa 9 @+DE+3) 9 9 FD+3) x ort =34[infe+ Uf ~ [Ine], =3+In4— In ~(In6~1n3) = 3+ In2 19 Solution 44x (oe (P+ D4-x bree +1 a,b,c constants. Then 1+4x 2a(x? +1) +(bx+c)(4—x). Equate coefficients of x7: O=a-b. Equate coefficients of x': 4=4b-c. Equate constant terms x°: I=a+4c Thus we get a=1,b=1,c=0. Hence eed: a4 tx 2 MN e=rarentea ease sept ita + Sine +f ~(n2~In4) +hans-in=In2+Fin5=51n20. 20 Solution Let wae “Sue o. ‘Then 85 (ax+b)(x? +9)+ (crt d)(x? +1). Equate coefficients of x7: O=atc. Equate coefficients of x7: O=b+d Equate coefficients of x1: 0=9a+e Equate constant terms: 8=9b-+d ‘SD Solutions Thus we get a=0, 4 8 Hence { ——2, 5 GIG? +9) n_m_Sn 3°18 18” TSD Solutions Series 2 Exercise 5.3 1 Solution Let «=x. Then x =u", dx = 2udu, and we get 1 du=2sin“! w+c=2sin\(x)+c. 2 Solution t Let w=x?. Then x= Vu, dx =—=du. Wu 3 Solution Let 2 = x41. Then x2u?-l,vr+1 =u, de=2udu. Hence JxVa-¥ lide = J(u? — rude = 2) (us — du 4 Solution Let u? =x-1. Then x=? +1, Vx , dx = 2udu Hence Jat VaerTae = fu? + yPududa=2f a8 + 2a uP \du=Zu? 42a 23 40 20.14 2420, pee Sat For - 1? 42 (x= 1 GOD Sa AS-D te 5 Solution ,u>0.Then x= Inu, dx=—du. u 1a 1 1 te) Nu = f*du-[— du =Inu- i( mai Jijau-JT yp ae= Ina Intute ‘SD Solutions Series | B =x-In(e* +1) +e. 6 Solution Let u=e8,u>0. Then x= Inu, dr= du u Hence x yeh +e ute I+u Steele a des Mau =ft* du= dut{—*5 dus tan“!u+ sin? +1)+6 J l+e™ Sea u rs ire ? fine Stan "e*) += ser +l)te. 7 Solution Let u=Vx,x20. Then x=u?, de = 2udu +l = udu =2} “5 du =2f lew? tu = Qu—2tan“"u)+¢ = 2vx -2tan (Vx) +e. 8 Solution Let x=sin?@. Since 0S x<1, we get 0s0<5, and dx =2sin 8cosOd0, 0=sin“"(yx). si 8 sin cosAd8 = 2] sin? 040 cos8 = ftd0— feos20d0 =0-.sin20+6=0-sindcos0-+e =sin"\(x)-yx(1-x) +¢ 9 Solution Let x= 4sin@ . Since -4< x <4,(x #0), we get — (8#0), and nls w n via dx = 4c0sd0, 0= on). ‘TSD Solutions Series 1s d= i st i tt aval He he (tan“!1—tan“!0) 14 Solution Let u=Vx,4,—m° = 2-2 o 19 Solution a 1 1 4 2tan(x/2) Let ¢=tan2,0¢x<%,0<1<—pe,x=2tan 1, de=—25 dt. Since sing = 20/2) 2 3 3 ie? 1+ tan?(x/2) we obtain #3 #3 ve 1 1 lee 2 dx = { —~———dr= = —at Ir S— Dian(x/2) ieee t= | eo e214 1+ tan?(x/2) 2 a — ws fit I=th 1-1/3 Be : 20 Solution x Let retnt, Ocx< Za 0crx sinx 1 1 Infsec x + tan x{ Fo te=sInjecx + tanx|+—seoxtanx +e. 2eos?x 2 2 12 Solution Jonsy%de=[xin?x} ~ fxn? xyde =e 2f rinx ax =e-2f nxde= 1 1 1 = 1 = e-Afrinx—a]})=¢-e-(e-1) = e-2 13 Solution oa ma Jxcosxds =[xsinaxf}?— fsinxdx =£+[eosx])” 0 a 14 Solution ap 172 Jsin.xcos.adx => Jasin 2xdx = o x2 {e cos 2x] - Froane| ° x) 1 +c, we get = fean™ ‘7SD Solutions Series 2B In=Jdx=x+e, J, = Jtanxdx =-In|oos2|+c. 16 Solution Let n 21. Integration by parts, with (In x)" as the second function, reduces the power of Inx. 2 = W dy = nlp ae ytd, un 1 1, = Jina) dx = nx)" 5 f xPn(in ay) de = (In xy" ~ 5 fx(inx) de 2 2 =Fanay" Fler» where Jp =f xdx 17 Solution Let n> 2. Integration by parts, with sin”“!x as the second function, reduces the power of sin x. 1, = sin" xdx =—cosxsin"“ =-cosxsin" | x+(n—1J sin"? xdx—(n—1)fsin” xde. x+ feosx(n—I)sin"”? xcos.xdx Hence (m-1) 1 an 1, =-—cosxsin”™! x4: n where Ip=[dr=x+c, 1, = Jsinxdx =-cosx+c. Let for n20 2 Jsin” xdx. 0 Then for n22 we get J, =-— m2 where Ig= [ar=2, ° 2 ‘SD Solutions Series 24 18 Solution Let n 22. Integration by parts yields 1 1, = sec” dx = [de = tanx = (n= 2)f tanx———sin xdx cos" x cos’ sx 1 1=cos* x = tan x—~ ~ (n- 2)[ — “ax cos"? x cos" x = tan.xsec””? x—(n—2)f sec" xdx + (n~2)] sec"? xd = tanxsec"™? x-(n-2)I, +(1-2)Ip-2. tanxsec™?x | n-2 Hence J, = 41,9, nl nad where Ip=Jdr=2+e, Jr iac=infsecx+tana|+e. cosx Let for n20 ms = Jsec" xdx. 0 sone Then Jean sect? xf" 42-2 7, nl nl me ms where Ip= fdr==, y= [secxde =[Infsecx + tan al] =In(v2 +1). o 4 a ° Thus we get 19 Solution Repeated application of integration by parts, with x" as the second function, removes powers of x from the integrand stepwise until the integral is known. Let n> 2, then mn in #2 1,= Jx"cosxde=[x"sinaf™ ~ fsinxnx™ "de 0 ° a oI m2 " -(£) talfoexarf? 0-0 for soens| (3) =n(n=DIq-2, where ‘SD Solutions Series | 25 a2 Ip= Joosadx =[sinxf? =1, 3 wa an 2 y= Jxcosxdx=[xsinx]})°— [sin xdx ° ° Thus we get 4 = _3n? +24, 16 6 15 4 2 —n* +90n* -720 64 8 20 Solution Integration by parts, with (1—x?)" as the second function, reduces the power of (I x3). Let n21, then 1 1 1 1,=fx-2)"de = +hea -ryl, 2 fn = B32 )ar = Sn fxd Bye 0 2 29 20 1 1 a3 nfl Py yar fx yd 2 0 2 0 i 3n. Hence J, =—2— 1, =I, with ioe 2+3n 2 Furthermore, for n > we have __3n_3n-3, 3 3n42 3n-1"?" 3n+2 3n—1 3n—4 = sn! © Gn+2)Gn=DK8-5-2" 7SD Solutions Series 26 Exercise 5.6 1 Solution (a)Let x -u, then du=—dx,x=0= w= a,x=a=u=0, and 4 ° 2 4 J f@dde=-J f(a-u)du =f f(a-u)du= J f(a—x)dx ° a ° o (b) 7 xsinx F(n—x)sin(n—x) , _ F(m—x)sinx Fsinx xsinx Joel ds = J de = nf de J ae T+ cos? x T+ c0s"(n— x 1+ cos? x Itcos*x gl+cos*x 0 a o 0 o Hence js _xsinx 4. =] sinx pltcos?x 2g 1+cos?x Using the substitution u=cos.x, sin.xdx = —du, x =O u=1,x=2=>u= 1, we get x 1 xsinx L ® == [du = nf —ydu = nftan™ uly =n ==, Seats ali ee re) 2 Solution (a) Using the following relations: j f(x)dx= J f(a—x)dx, ° o ( ) 2. _ 1+c0s2x 0s 2x sin| £—x]=cosx, cos? x= =, sin swe get “lsinx a, a= in[2(e/4—2)) = [ea ="f'sint Bp. 9 Itsin2x 9 1+sin[2(x/4—x)] a 1+cos2x 9 cos?x m4 ws ms ="Ton?a-'f( ec J ae Jae =[tan xf z. 0 9 \cos? x 9 cos? x i 4 (b) Using the relations, f f(x)dx =f f(a-x)dx, o a 7 cos? x i= T_ cos*(n/2~ x) fara sings! 9 cosxtsin’x 9 cos'(x/2—x)+sinX(n/2-x) 4 sin?x+cos?x Hence ‘SD Solutions Series 2 aa" itt tea f tt gpa" cotta “fae a er © 2 Thus 1=2, 4 3 Solution (a) Using the substitution u = —x, we have j Fooe=-| ‘Sf (-u)du= scandal ‘f(-x)dr. Then f ponae=f sonde+ fronde=firons semiae. ea ° es ° (b) mm aD J sin? xdx = J fain? xe = aaalte™ Ite" Ite “Ths in? oe cata} den t= Z (ef +)dr= ‘fant str=3 ‘fo cos2x}dx ° in?(-x) de m = a Jeosaaas=% =4 4 Solution (a) Using the relation freode= fis Sa ))de, we get a a 1 Ife 1 at { i! Jic-i{ +a = fide a1. olite™ iret glet+l Ite’ a wey mary f 1 1 eg J—«- J + ox J— aal tin Trsinx* ey {ream I=sinxJ” 9 1-sin?x me =2) —y;-4r=2 =2. [yaa 2linsh* ‘7SD Solutions Series 28 Diagnostic test 5 1 Solution If f(x)=Inx, then f/(x)=—, and the given integral follows the pattern Jf@OF ax =i sere Hence [2 ae = y(n xPte. 2 Solution If f(x)=—, then f(x) and the given integral follows the pattern x x : : . flO pade= ef +e. Hence fel’ dr =-e"* +0. x 3 Solution If f(x)=sinx +2, then f’(x)=cosx, and the given integral follows the pattern cosx 2+sinx JP'S’ de = In] f+. Hence f dx = In2+sinx|+c=In(2+sinx) +c, since 2+sinx21 4 Solution If f(x)=x2 +1, then f’(x)=2x, and using the pattern [pans Fa+e, wD 2 y sP we get pegte= [iw ] 5-1 es 0 5 Solution In(a?+9)+dtan" Zc. 6 Solution tT 8g b,¢ constants (DOH) xD x4 " ‘78D Solutions Series 29 ‘Then x+7 a(x +3)+0(x-1). Putx=1: 8=4a=9a=2. Put x=-3: 4=-4b=9b=-1. x47 1 1 Hence Sera? =f ode ft a= Biri +3} 7 Solution 2x? 2x41 Let ———— = (x-2)07 +1) x-2 a, b,c constants. n 2x? ~2x+1 = a(x? +1)+(bx+e)(x-2). Putx=2 S=Sa=a Equate coefficients of x”: 2 Equate coefficients of x1: -2=-2b+c=>e=0. Hence 2x? 2x41 { _— (2D? +1) =In(x—Wa? +1) +e. x 1 x ee el ea = Sgt gear gine +)+e 8 Solution 4 ha axtay sq ty 4 4 4 44 [rte 5 Jn = f(a - 2 1) xe = J 28d -2f nde + [de 2 * * 2 2 2 2* " ‘ (2 ~[P ff final = 64 4-16-4444 ~In2= 48-4102. b 9 Solution Using the substitution u= Vx, x= u?, dx = 2udu, we have 1 u 1 “1 = = J Ft = oJ au = 2tan! ete = 2tan"/x) +e. Wak ae “ Te “ a ww TSD Solutions Series 30 10 Solution dx = 2udu, we have Using the substitution u? = x+1, x =u? 2 udu = 2 (u? = 1du = = 2+ %-2 k+1+e baked 11 Solution d9,x=2-=0=4, x= 2 36= Using the substitution x = sec®, dx = 5 cos? 3 4 » we have cos® na 1 oe —— -tanOd@=[8] =n/12. ine “i (1/cos?@)=1 cos? i tan, hee 12 -Solution Using the substitution 7 an pxsO=ta0,x= 5a rahx= tans, de= It, we get 1+? m2 4 1 4 2 — ae = J, Trees 9, , 1? ler dt = fldt=1. ears 13. Solution Using the substitution w= sin x, du =cosxdx , Jwsinx cos? xdx = f-VSinx(1—sin® x)cos.xdx = [Jul u? du = f Vudu — [udu 2) 2 8? Hy? sin’? x40. +c= 14 Solution Using the formula 2sin pcosq in pq) +sin(p+q), we get Jsin4xcos xdx = +Join 3xdx +4 fsin Sxdx foosax— a cossx +e. ‘78D Solutions Serie 31 15 Solution Integration by parts, with x? as the second function, removes powers of x from the integrand fxtetde = etx? -2f xed = o%x? af xe" —fetdr} = x7e* -2{xe*—e"} +e =xte% —2xe* +2e* +. 16 Solution Integration by parts, with x as the second function, removes powers of x from the integrand fxeosaude =2xsin2x—4 fsindxde = 17 Solution 18 Solution Let n21, and (nzy"de =[x(In oy - f(an x)") deme —f mn xy"! 4 d= e-nfin xP ld 1 1 1 Senmy-ty where I= fide =e-1. 1 Hence I, =e-41; =e-4(e-3,) =—3e + 12(e-2h) = =92-24. Je - 2A(e- Ip) = -15e+24(e~1) 19 Solution Itis clear that f(x) = x®sin x is an odd function, since F(-x) =(-x)® sin(—x) = -28 sin x = f(x). Hence #2 JxSsinxdx=0, oa because of the fact that ‘SD Solutions Series 32 fraae Jiveo+s-n}ax=0 if f(x) is odd. co a 20 Solution Using the substitution w= a—x, du=—dr, x =0=> u=a,x=a=>u=0, we get 4 ® @ @ Jf @)de=-] f(a-u)du= J f(a-u)du= J f(a-x)de. a a ° ° Hence A m2 cosx 7 cos(/2-x) 7 sinx = f[ a= = {Sty 9 cosxtsinx — 4 cos(t/2—x)+sin(a/2—x) 4 sinx+cosx ” since cos(/2~x)=sinx, sin(/2~x)=cosx. Then a2 x2 x12 i m2 12 cosx sinx cosx+sinx a= j ———_atfo des [ SS ar= flar=2. p cosxtsinx 9 Sinx+cosx” g cosxtsinx. 92 x Hence J/=— 4 ‘SD Solutions Serie 3 Further questions 5 1 Solution Using the pattern f f(x) f’(x)de =f Pte with (x)= tan“! x, we have tan“! x +x? de = Han late. 2 Solution Using the pattern J— +c with u=x+2,a=1, we get 1 lu= x4 atulteent 2" |url| 2 =) tae = far = eaxe3*(x+2)?=1 3 Solution Using the substitution e* +1=u, du = e*dx, we have = Inju|+c=In(e* +1) +c. as 4 Ke +I) +e 4 Solution ata +e with u=x+2,4=1, we get ar ge = J} Ate = [n= tan Fate tan 4 2)-40. x 4x45 (r+2)P +1 e+] 5 Solution Integration by parts leads to a more simple integral In(x? = Idx = xIn(x? ~1)- J eal =xIn(x?=1)— AS d= xin(x? =1)-2{de-2f. ‘SD Solutions Series 34 7 Solution [sin SxeosSxdx = [(6in2x +-singxpde = 2 fsin ad +5 fsinBad = ~Leos2x~cL cosBx +c 8 Solution Using the substitution x—1=u and the pattern [ — HbsAeeswe have um 2a |utal Jae = J yas =~ te =f cu = tle 342x—x 4—(x-1) (1-4 wa4 ur es ett 4 |[x-3| 9 Solution Using the substitution e* =u, du=e%dr, we get de = fd = tan Mute =tante* +c. +1 w+ 10 Solution Integration by parts leads to a more simple integral Jin(x? + dx = xin(a? +1) fx(in(x?-+ Ide = xin(x? +1)— [2 —2de x +] 241 1 41 = ringx? +1)-2f = xin(x? +1) -2f de+2{ Lae = xin? 41)-204 2tanlte 41 11. Solution sinx J(tan.x+cotx)dr = f cosx dx + [Ade = cosr / sinx In|cos x|+Infsin x|+¢= an | +e 12 Solution Using the substitution w= 1 “Ke —1 and the pattern =sia”’ +c with a=2, we get Ny uP 7 § 1 = L jl inl (x1 = = sin <+e=sin | |4c. ae coe 4—(x—1) 4 — “ee ers 7 are el 74 2 ( 2 ) 7SD Solutions Series 35 13 Solution Jsin4xsin dads = 5 f(00s2x—cos6x)dx = 3 {cos 2nd ~ 5 Jeos6xdx =Asinax-Esinér+e. 14 Solution 2 Jae = ppt = ide + fotos rednctee xe net, x-l 2 |x-]] 18 Solution Using the substitution x = 2sin u, dx = 2cosudu, we get Jl4=22ae = Jf4i—sin? u)2cosudu = 4) cos? udu = 2{(1+c0s 2u)du= 2u+ sin 2u+c z =2sin!= +2sin ucosu+¢ = 2sin"!= + x) te=2sin 242 f4—-x? +0, 7 2 4 22 16 Solution Using the substitution u=In x, du=—dx , we get x 1 ec?(In x)dx = sec” udu = Jed =tanu+e=tan(lnx)+e. cos? 17 Solution 2 Using the substitution t= tan, cosx = x= 2a", dx = 2 de, we have 1+?" 1+? 1 1 dx = | ——__.——___ — dt = 2] —dt= Ser i (=P 14H) 141? Pe Freatzte. 18 Solution Make the substitution u=.x +1. Hence 2: tet ag = fe) den [du = 2} de fta 42x42 7 (xt +1 wet ws =n? +1)- tan +c = In(x? +2x+2)-tan (xt) +e. ‘SD Solutions Series 36 19 Solution Make the substitution cosx = u, du= Joi sin xdx . Then we have 3 xe0s? xdx = f (cos x—1)cos? x(—sin x)dx = f(u? = I?dus fubdu-fdu _c0s'x _ cos? x 5 3 +e. 20 Solution Integration by parts leads to the following relation Y 2 T= [Vi6+x7dx = xV16+x? -Ja{vi6+*) dx = xV16+.x? -J- ae Vi6+x7 a rVien at JEM g, « pfigaa® - [Vea t6dr +16{—— xV16 +2 aa xV16+2x7 — J Vx? +16dx + IT = x64 x? ~ 1+16{ pa nr "Tot Hence fvl6+x7ax plier? +8[qchgee Sito +x? +8in(r+Vi6rx) +0. 21 Solution eins ‘ Using the pattem fel) f/(x)dx =e! +c, we get Soper The 2 x 22 Solution 3x?-6x+1 (x-3(27 +1) x-3 x? 41 bxte Let + @,b,c constants. Then 3x? ~6x+1= a(x? +1) + (bx +e)(x-3) Put x=3: 10=10a=a=1. Equate coefficients of x7: 3=a+b=>b=2. Equate constant terms: 1=a—3e=9¢=0 Hence 7SD Solutions Serie 37 2 [af ine —J+In(x? +1) +e (x-37 +1) =In(z-4-(2? +) +e. 23 Solution Make the substitution x~1 =u", dx = 2udu Then 1 1 lea a —t aj et a ig ae 2 u - =2J— du 2A Cay peit= Pea tut] 1 ty al are ut+——-J—du w+) = 2tan yt A— tan ute = tan a+ ny tes tan "x =1)+- w+ i 24 Solution 2x?-x+20 (2-2)? +9) Then 2x? - x+20= a(x? +9)+ (bx +ex—2). Putx=2: 26=13a=0=2. Let a,b,c constants. Equate coefficients of x7: 2=a+b=>b=0. Equate constant terms: 20=9a~2e=> ¢ = Hence 2x?-x+20 EE dx = afi a Sea 5} te 2% at Wy We +9 = 2Infx— = Fran! 25 Solution Make the substitution e* = u, du = e*dx Then e 1 _ ome 1 ute=sin"(e*)+c. ‘SD Solutions Series 38 26 Solution 12 axth | oxtd + 1+, «= , a,b,c constants, (7 4427 +16) x? 44 x? $16 Then 12 = (ax-+b)(2? +16) + (crt dy(x? +4). Equate coefficients of x? ate, Equate coefficients of x7: O=b+d Equate coefficients of x1: 0=16a+4c. Equate constant terms: 12=16b+4d Thus a=c=0, b=1, d. Neen (Se eld (x? +4)(x7 +16) x4 x7 +16 x44 sand 1 +16 27 Solution Make the substitution cos.x = u, du=—sin xdx Then we have a 2 2 Sn ae = {eos x=I) (—sinx)dx = ai = = fldu- sa = 4, Teas J cos? x ‘ » J we i j Sz a wr “ =cosx + +c=cosx+cosecx+c. osx 28 Solution Make the substitution Ay a,b, ¢ constant. Then 1 (1? +1) +(bt + eM +1). Put x= 2=2a=a=1 Equate coefficients of 17: O=a+b=>b= ‘FSD Solutions Serie 9 Equate constant terms: 1 Hence Wm tanx “e-aak ia -| aa fine Viana 1+t ol+ ate=c=0. 29 Solution jet axel fino? +f, fants] =tin2+, “hie x +l “igh x41 30 Solution Itis clear that m2 T= [i¥sin2xdx = 0 I [iFsinwae= [firsinzae. Make the substitution sin x =u, x= . x ade = du, x= 09 u=0,x=2=5u=1. View 2 Hens = | | < u=—2[Vi=u] =2. 31 Solution 4, 2 e4x-20 a dete (xt 2x? +4) x42 x2 44 » @, b,c constants. Then 5x?+4x—20= a(x? +4)+(bx +c)(x +2). Put x= =8a>a= Equate coefficients of x7:5=a+b-2 b=6. Equate constant terms: -20= 4a+2e => Hence j $x2+4x-20 hdr = - ze 6 wate 8 ja setae) Jee fe “2 peed -In4+In2+3In8-3in4—=2In2—z. ‘TSD Solutions Series 40 32 Solution x Using the substitution r= tan, and r=, dt, x20 1=0,x 2 sinx =——5, cosx = +r? l+e 1+e Tene a dt 3cosx+4sinx+5 ite a a 2 +8148 g(t +2)* 33 Solution . 3x? - ay b Let 4" (x-2alx? +a") x-2a Then 3x? — ax = B(x? +4?) + (ext d)(x— Put x=2a: 10a =Sa"b=>b=2. Equate coefficients of x?:3=b+c=> c= 1. Equate constant terms: 0 = ba’ ~2ad => d=a. Hence j 3x2 - ales xta o(r- ae Fea gl*-2a x? +? =o) art [a f+ gine? + 9p + ftan-txrale o 0 an 3 -2n2+5In24 —=1n2. 2 34 Solution Let n be positive integer. Then [cosnxdx 2 {sin nf, = Losingn— sing) 0, ifn =2m+2, sinEn=|—+, ifn=4m+l, where n 0,1, 2,3,.K, 1 itn=4m+3, n SD Solutions Serie 41 35 Solution Since 2cos pcosq =cas(p~q)+cos(p +4), we get Cos mxcos nx = Ffeoscm- nx + cos(m-+ n)x} 2n Hence I 1 Let men, th ee 2m+n) [sin(m-+ndx fF = Let m=n, then 1=420-+— {sin 2mxP* = 4m a » since [dx=2r ° 36 Solution 1 sectx 4 tan?x Let us show that ———~ =: — 9-8sin?x 9+tan? x A+tan? x 1 t 1 -p-8 we] -[pesistes] _lttan?x __ sec?x 9-8sin? x Vetan?x I+ tan? x O+tan?x 9+tan?x” Hence, using the substitution w= tan.x, du= sec? xdr, x= 0-9 u=0,2=F9u=V3, a HB sec? 8 wehave f —'ax='f $2 * a= f 1 d= eat Jaan n}e- 0 9-8sin?x = g O+tan?x 9g 4 3 3) 3. (3) 18 37 Solution 2 Let us show that SC ¥— Since sin? x= We get +tan?x i rt — tan? 1 2 2 -p- tan? x ] i: tan ] Litan?x __ sec?x 9-10sin?x 9—tan?x 9-10sin? x L+ tan? x Y+tan?x | 9-tan?x 9—tan?x” : _ 2 x Hence, using the substitution w= tan, du=sec? zd, x=O=9u=0,2= 29 u= 3, we have ‘78D Solutions Series 1a 6 3. V3 eet P-cBy 38 Solution Make the substitution. x = Ssin?@ + cos” @=4sin?@+1. Then dx = 8sin@cos6d0, r x x=230=5,x54— 4 "6 = 15 —Ssin?@—cos?® since x=4sin?0+1. Hence f J2—* = oo en OH See sin cos8dd gVx=1 ggV sin? 0-+cos" 0-1 m3 3 n@.cosOdO =8 fos? ed0=4 {(1+c0s20)d0 m6 6 ae 2 2 = 5nd foosted0 = Fn+2fsin20), e=5e 116 39 Solution Using the substitution x = Ssin?@ + cos” @=4sin70+1, we get dx = 8sinOcos6d0, x=230= F since x= 4sin?0+1. Hence i 8sin8cos® 47, ro x “le [San orcas? 8—1NS—Suint@ os") sin@cos6 =4 = fdao= Temes i 40 Solution Let n=0, then integration by parts leads to Jinxde = xinx~ J x(Inx)de = xInx-fde=x(Inx—I)+e. Let n=1, then [RE ac=tint xtc. z Let #0 or I, then integration by parts leads to ‘7SD Solutions Serie 4a 41 Solution (2) Using the substitution u= x? +1, du=2xdx, we get J xo (a 1 a +1 G+? 2u” au 1 1 — +e. 2a aa? en?“ (b) Using the substitution x= tan, we have 3 tan?@ c? @d8 “\caton oe sec%8 aa oi) + te 4 Tax? 4 2 14x? 4427)? ‘This is in complete agreement with result derived in (a). 42 Solution (a) Make the substitution x= asec@. Then ¥x*—a? = atan@, and dr =a = cos! Hence Near = 0? fan Si8 gg = g2f!=608" 04g = 2! at cos? cos’ cos*@ cos® =a? sec? ad0— a? seca. Using the recurrence formula jseo”"*! xdx. ‘SD Solutions Series This way, we get 2 2 J [Pade = 75 InfsecO + tan|re= 2 Pa? -S nfreVx?— a7] +0. cos (’b) Integration by parts yields 7 2 T=J Pa Pac alta? —f( Va") dx =e? yea . Ix? a’ 2_ gag? = 1 axl? -a? Pent ? ~ IMac aba 08 f Pores xa EF ; 2 Hence 1=>xVx?—a? -“f 2 2 Te 43 Solution | 12 Let 1,= J snare, n20. Since sin(2n-+1)8—sin(2n—1)0 = 2cos2n0sin®, > sin we get for n21 m2 m2 = 2 { £082n0sinO 9 - 2'fcos2nade = +[sin 2nd]e” =0, 3 sine a n 44 Solution wt Let I,= f seen FP 9, n20. Since cos(2n +1)0 + cos(2n—1)8 = 2cos2n8cos0, cos! 0 we get for n21 2 mn Int Ina =2f 2082180080 9 — 9"fcos2n0d0 = 1 fsinano fe” =0. a) i n x2 _ Hence f, =—,, and f, = (-1)"fp =(-1)™ fao=(-"5 : 3 45 Solution Integration by parts yields for n 21 SD Solutions Serie 45 = fede = [a fae? 22") de = 2 = Py ae H Q 0 = nf (x? — a? +a? )(a? = x?"Mdx = ~2nf (a? — x? hdr + 2na? f(a? — x?) de ° a ° =-2nl, +2na7Iy 4. 46 Solution Let for n20 xd 1, = Jsin® xcos* xdx. a ‘Then integration by pasts yields for 22 mn : ae a 1, 1,= [sin® x00s? xd =—_[oosxsin™! x] fin" adcosx=—1— fsin™? xde. i ne nei} te It is clear that xd fsin" xdx for n22. a 2 m2 On the other hand J, = fsin”*? xde <4 Join" x0.-cos? x)dx ° ° nevi 1 1 =—— fain” xdx-—— sin" xo0s? xdx = ntl n+l 4 n=l ele nel? n+l Hence nt2_nol n=l wet mt et In yg len nee 3 aa ae (2, #2 Furthermore 14= fsin‘ xcos?xdx == y= 2-7 ly = foos*xdr=7-| [ldx+ Joosdxde ° o 8 4 64° 8 1(£ Minas? l= # ~i5 rain \ 47 Solution Repeated application of integration by parts can be used to reduce the power of x. in the integrand stepwise in the following way: ‘SD Solutions Series 46 2 ai Ri 1, = [x sinadx =—foosx-x"[" +n [oosx-x" ° ° : a2 1 7? _ n(n=1) [sin x-x"-2dx oO for 22. Hence HI? m2 ny ny « we hs [tsinxix=4(2) -12h =) -1{24-219) = 121 +24 fsinxde 4 2 2 2 2 a we 2 12n +24, 48 Solution Repeated application of integration by parts to reduce the power of cos” x in the integrand stepwise leads to the following: a2 ni ni? Iq = feos" xdx =[sin.x-cos"'x]"" +(n=1) feos"? x-sin? ade ° o m2 m2 m2 =(n=1) Joos"? x-(1—cos*.x)dx = (n=) feos"? xdx—(n—1) feos" xdx 3 ° ° = (n-Dly-2-(n- DI, Hence for n>2 n= WIyag > ty =, a Furthermore, we get x2 m2 683 Joos! udu= feos! udu=Iy=——m. anid m2 o 512 49 Solution Make the substitution « =f - Then x, © 6 7 n2 6 aft nid wz Sle-F cos? xdx=— f (-u)® cos’ (E-ua= Jud sin? udu= fxSsin? xdx =0, 0 Wi 2 wi? —w2 ‘SD Solutions Serie 47 since x°sin? x is odd. 50 Solution @ on Letus show that f f(x)dx= [{f(x)+ f(a—x)}de. o ° 4 an * Itis easily seen that f“\<)dx = [ f(x)dx+ J f(x)de. Make the substitution x=a—u inthe a a on second integral. Hence « a on a2 on Jfldde= J foddx- fla-uddu= [f(adde+ J f(a-uddu= [{f(x)+ fla-x)}dr. a ° ef ° ° ° Using this relation, we get * m m2 Jasin xde= f [xsin': +(n—x)sin6(n—x)}de = f {xsin®x+n sin’ x—axsin® x}as ® 0 a 2 a =n [sin® xdx ==, ° a since a2 D a2 1, = J sin" xdx =-fsin’” xeosx] +(n=1) [sin"? xc0s? xde ° ° m2 =(n=1) fsin™? x(l-en x)dr = (n= Up-2- Iq)» ° 7SD Solutions Series 7SD Solutions Series Worked Solutions to Popular Mathematics Texts Suggested Worked Solutions to “4 Unit Mathematics’’ ( Text book for the NSW HSC by D. Arnold and G. Arnold ) Chapter 6 Volumes HARE S HARI, 971097 J § eB & § Yor co” COFFS HARBOUR SENIOR COLLEGE i Solutions prepared by: Michael M. Yastreboff and Dr Victor V. Zalipaev 7SD Solutions Series Worked Solutions to Popular Mathematics Texts INDEX page Exercise 6.1 1 Exercise 6.2 8 Exercise 6.3 15 Diagnostic test 6 . 4 Further Questions 6 29 Solutions are to “4 Unit Mathematics” [ by D. Armold and G. Amold (1993), ISBN 0 340 54335 3] Created and Distributed by: 7SD (Information Services) ABN: 13009821 © 780 1997 The 7SD team welcomes all feedback. Correspondence should be addressed to: 7SD attn: Michael Yastreboff PO Box 123 Kensington NSW 2033 Exercise 6.1 1 Solution a) Aslice taken perpendicular to the axis of rotation is a disk of thickness &x and radius r(x)= x7. The slice has volume 8V = nx“ . 1 1 | im S axtix = frxtde =| 2% : Ve pdm < | tar 22] 7 the volume of the solid is 7 cubic units. b) A stice taken perpendicular to the axis of rotation is a disk of thickness 8y and radius r(y)=1-y. The slice has volume av =n(1- yo). cubic units. ©) Astlice taken perpendicular to the axis of rotation is an annulus of thickness 8y with radii 5(y)=y and 7, =1. The slice has vohurme SD Solutions Series m the volume of the solid is 5 cubic units. y* ) A slice taken perpendicular to the axis of 2 rotation is an annulus of thickness 6x with radii 4(x)=1~x? and ry, =1. The slice has volume (2-8 r=[1- (1-2) for = nar? 2") bVe : 7 tthe volume of the solid is a cubic units, 2 Solution a) Aslice taken perpendicular to the axis of rotation is an annulus of thickness 8x with radii n(x)=4—2? and 1, =4. The slice has volume BV =a? = )Be = (8x? — pee v =n Senor - +x = [x00 Ja a2 24m 2 the volume of the solid is cubic units. ‘SD Solutions Series b) A slice taken perpendicular to the axis of. rotation is an annulus of thickness 8y with radii n(y)=4= y and 7, =2. The slice has volume By =a(eh-9)= [2° -(VE=7) fy = mo VS = |miy-n2| =e the volume of the solid is 8% cubic units. ©) A slice taken perpendicular to the axis of rotation is a disk of thickness 6y and radius r(y)=2—./4= y.. The slice has volume BV = n(2- 4-9) by = 2(8-y-4/4- yy im 6- y-aJIm9)o= fae 7-7 Substitution y=4-y’, dy=—dy’ gives = + ad dy’ eal aye 49 Y 2 88 va-faleey-457)= oltre r vy i the volume of the solid is = cubic units. 4) A slice taken perpendicular to the axis of rotation is 8 = a disk of thickness &x and radius r(x)= x". The slice ey has volume 8V = mx‘8x. 2 2 fae fea oe V= fg Deai'be = [mae = ak the volume of the solid is ae cubic units. TSD Solutions Series 3 Solution Y4y A slice taken perpendicular to the axis of sotation is an annulus of thickness 8y with radii i(y), ra(v). where 4(y)>(y) and (9), n(x) are the roots of y= 2r—r? considered as 1 yode-x? a quadratic equation. The slice has volume BV =n tinny. P-2rty=0 na alF ath non=2fimy 8V =4nJi- y by V = lim) Anyi y&y = [anim yay 0 a Substitution y y’, dy=-dy’ gives ° : Vad] y= a2" '. the volume of the solid is = cubic units. 4 Solution Yt, A slice taken perpendicular to the axis of rotation is an annulus of thickness 5y with radii (x). a(y), where 4 (y)>n(y) and 5G), 1 (y) are the roots of y= 6x 4x" = 6r* ~r* considered as a biquadratic equation. The slice has volume bv =n(7—7 fy. ya6r? ‘SD Solutions Series = Sse oe= frst ste | SEP x the volume of the solid is 36m cubic units. 5 Solution A slice taken perpendicular to the axis of rotation is a disk of thickness 6x and radius r(x)= sii The slice has volume BV = nr(x)Bx = nsin? xBx. cos 2x the volume of the solid is oe cubic units, 6 Solution Astice taken perpendicular to the axis of rotation is an annulus of thickness 6x with radii 4(x)=1—cosx and 7, =1. The slice has volume BY = n(P -12)8x = n(2c0sx—cos? x)Sx. V =n Simones cos se fresco = 7SD Solutions Series =2n-=, = [n{2oosx— 1490828 as nn{2sina 2 2) } 2 2 4 2 . tr the volume of the solid is 2x“ cubic units. 7 Solution Y4, A slice taken perpendicular to the axis of rotation is an annulus of thickness 8y with radii %(y), »(y), where %(y)>%(y) and (y), n(y) are the roots of (r—1)' + y? =1 considered as a quadgatic equation, The slice has volume BV =a, +X — 5B (r-1f+y?=1 P-ar+yte0 v= lim S Ary => by = [any i= yay. Substitution y=sing, dy =cos@do gives 2 2 2 Vaan ffi F9 cospdig=4n f cos*pdp=an | +5529 gy oe? wz nw? 2 - ar(or ze) oon? 2 lw +. the volume of the solid is 2n* cubic units, ‘SD Solutions Series 8 Solution A slice taken perpendicular to the axis of rotation is an annulus of thickness 8y with radii 5(y), 7 (y), where 3(y)>(y) and (y), 5 (9) are the roots of (r= + = considered as a quadratic equation. The slice has volume av =n( +5 e-n)By- > aed (1) +5 Pore 4 ntn=2 7 n-R=2fI- Savy BV = 2nJ/4— y* By. 2 2 V = [im 9) 2nf4— y* By = fam ¥* dy. mt a Substitution y=2sing, dy=2cospdg gives ” 2 ” Vaan | 4—4sin®9 cospdp=8n | cos*gdg=8n f Hee820 ap “a wn -m wan(or S22) ca. 2 wi? the volume of the solid is 4n* cubic units. ‘SD Solutions Series Exercise 6.2 1 Solution a) The typical cylindrical shell has radii y, y+8y, and height A(y)=1—./y This shell has volume BV = al(y+ 9) ~y*]h(y)=2n(1- Vy) By (ignoring (8y)"), ma San( -Vo po =O) a ysay! arf 22 & 3/2), the volume of the solid is a cubic units. 'b) The typical cylindrical shell has radii n(x)=1-x, {x)= 1-x+ dx, and height h(x) = x?. This shell has volume BV =nf(i-x+8x) ~(-af]AG) = 2nx*(1- x)8x Gznoring ()). v i x)8 = alee -x)de= ofS xy a the volume of the solid is t cubic units. Yt c) The typical cylindrical shell has radii x, x+8x, and height A(x)= x7. This shell has, volume ax) By = af(e + Bx) — 2] (a) = 200 > (ignoring (8x)'). Ve ii oa jim 2 Be = | x° de 7SD Solutions Series the volume of the solid is ; cubic units. 4) The typical cylindrical shell has radii 5()=1- y, n(y)=1- y+ dy, and height h(y)=1-4fy.. This shell has volume av =a{(- y+8))-(-») A) =2n(1- y1-Jy)8y ignoring (6y)'). v = ym Yont- y\i-yy)by =f fi-W7} 2 Lo yn =28[(-9 -yeylapern[y 2 = the volume of the solid is z cubic units. 2 Solution a) The typical cylindrical shell has radii y, y+ Sy, and height A(y)=2—.f4—y . This shell has volume av = af(y+8y)—y*]H(y)=2n(2- fay) yay (ignoring (6y)"). Y= hm S2n(2—J=9) 98 =2n] (0-5) yay. Substitution y=4~ y’, dy=—dy’ gives Ve -2nf- ya-dy')ay’ = anf(—4y"" ~2y'+y™)dy" 4 0 : Dl er =ar(ay “3. y Zi- BO the volume of the solid is Be cubic units. ‘78D Solutions Series b) The typical cylindrical shell has radii x, x+8x, and height h(x)=x*. This shell has volume By = a(x + x) = Jae (ignoring (6x)*), He) Ve im Y dns? &x= 26] Cae the volume of the solid is 8 cubic units. x=2 ) The typical cylindrical shell has radii i n(x)=2-x, n{2)=2-x-+8r, and height A(x) = x*. This shell has volume av = nfQ2—x+8r) -(2- a} = 2nx*(2-x)Bx ignoring (8x) ). V = jim Y2nx?(2~ x) = 2 x x : =2n[7(2- =2n| 2=--= 2x] (2-x)dx n(28 | > id ig 8% the volume of the solid is —* cubie units, 4) The typical cylindrical shell has radii 5(y)=4- y, n(y)=4- y+8y, and height h(y)=2-J4—y. This shell has volume av = Al(4— y+8y)'-(4- ya) =2n(4- v2 —f4=y" ey Ggnoring (Sy) ). v= lin, San y2-J4=y)By Substitution y=4~y’, dy=~dy’ gives Ve -2nf y¥Q ~y)ay’ = ai(2 the volume of the solid is = cubic units. TSD Solutions Series 3 Solution The typical cylindrical shell has radii x, x+6x, and height A(x)=2x—-x?. This shell has volume 2 . > fg warfare \eram(2 5-2) = the volume of the solid is = cubic units. 4 Solution ‘The typical cylindrical shell has radii x, x +x, and height h(x)= 6x? - x*. This shell has volume BV = f(x Bx) 2°] a(x) = 2ex(6x* — x4) Bx ignoring (8x) ). % 2_ 4 x i eee -atjerer(o =] = 36m. the volume of the solid is 367 cubic units. TSD Solutions Series 5 Solution The typical cylindrical shell has radii x, x+8x, and height h(x)=sin x. This shell has volume BV =n[(x+ x) — 2 Jax) = 2mxsin x8 (ignoring (8x)'), 2 a ss Y= fim) 2resins Se = 2 J xsinxde =-2n | xdoosx ae fi corns} =anin = ° the volume of the solid is 2m cubic units. 6 Solution Yy ‘The typical cylindrical shell has radii i@e)=5- x. (x)= 2—x+8r, and height H(x)=cosx This shell has volume bv= AG + wy -E J fe =n $x cosxae (ignoring (8x) ). v = fg r( $s pos = 28 f (E-xJeosnas. Substitution x . dx =—de’ gives = 2nsinx'” the volume of the solid is 2m cubic units. ‘78D Solutions Series 1B 7 Solution Yt The typical cylindrical shell has radii Is x, x+68x. Height of the shell is obtained from (e-1f+yt=1 ysi-G-ly HQx)=2y =2y1-(x-1). X The shell has volume BY =af(x-+ 8x) ~ x )hG@) = 4ney1—(x=1) 8x a (ignoring (8x)*) v =i ten I= (ef emf afi (=f a Substitution x= 2’ +1, de= dx’ gives 1 1 \ V san (x’+1Wi—x? de’ =4n) xix? dx’ anf Vix? dy’. ‘The first integral is equal to zero since the integrand is odd. Substitution x’ =sin, dx’ = cos@d@ into the second integral gives 2 ” ” v=4n [Ji-sin?@ cosqdp=4n | cos*gdg=4n [ uscos20r -n - ‘in 2 = ar( 0 + 2s] =2n" the volume of the solid is 2x? cubic units. ‘7SD Solutions Series 4 8 Solution y+ The typical cylindrical shell has radii x, x +x. Height of the shell is obtained from A(x)=2y =4y1-(x-1). The shell has volume av = af(x+ 8x) — Jae) = 8nxyi-(x-1) dx (ignoring (Sx). Substitution x= x'+1, dx= dx’ gives V=8n] (x +1Wi—x7 de’ = 8x x/VI—x7 de’ +80 VIX? ae’. The first integral is equal to zero since the integrand is odd. Substitution x’ = sing, dx’ = cos@ dg into the second integral gives V=8r fimare 9 cospdp=8x 7 cos’ pd =8n >= aba tap r(h-z aD 7 h ‘SD Solutions Series 18 The slice has volume av

También podría gustarte